SlideShare a Scribd company logo
Luyện thi học sinh giỏi, thi chuyên toán lớp 10 Thầy Hồng Trí Quang
1
Nội dung
Phần 1. Kiến thức bổ sung
1.Số đo cung – góc ở tâm – góc nội tiếp
2.Góc ở trong và ngoài đường tròn
3.Góc tạo bởi tiếp tuyến và dây cung
Phần 2. Chứng minh tứ giác nội tiếp
Dấu hiệu 1. Tổng hai góc đối bằng 1800
Dấu hiệu 2. Hai góc cùng nhìn một cung bằng nhau
Dấu hiệu 3. Dấu hiệu tích.
Phần 3. Ứng dụng tứ giác nội tiếp
Ứng dụng hai hệ thức
Định lí P tô lê mê
Chứng minh ba điểm thẳng hàng
Chứng minh ba đường đồng quy.
Tài liệu này của khóa học “Luyện thi học sinh giỏi, thi chuyên toán lớp 10” của thầy Hồng Trí
Quang
Facebook thảo luận https://www.facebook.com/chuyentoanlop9/?ref=bookmarks
SỐ ĐO CUNG – GÓC Ở TÂM – GÓC NỘI TIẾP
Bài 1. Cho tam giác ABC nội tiếp (O). Tia phân giác góc A cắt BC tại F, cắt (O) tại E. Chứng
minh:
1/ Tam giác BEC cân
2/ BEC ABC ACB 
3/ . .AB AC AE AF
4/ 2
. .AF AB AC BF CE 
HD: Bài đơn giản, câu 3, 4 sử dụng tam giác đồng dạng.
Bài 2. Cho đường tròn (O) tâm O, đường kính AB = 2R. Gọi PQ là một dây thay đổi của
đường tròn (O) sao cho PQ = R. Vẽ hình bình hành PAQM.
Luyện thi học sinh giỏi, thi chuyên toán lớp 10 Thầy Hồng Trí Quang
2
a) Chứng minh rằng B là trực tâm tam giác MPQ
b) Tính theo R khoảng cách từ tâm O đến PQ
c) Khi dây PQ thay đổi thì điểm M di động trên đường nào?
HD c) gọi H là giao AM và PQ, thì OH là đường trung bình. Điểm M di động trên đường tròn
tâm B bán kính Rsqrt3
Bài 3. Điểm M tùy ý trên đoạn AB cố định. Trên AM và MB dựng về một phía đối với AB
hai hình vuông. Các đường tròn ngoại tiếp các hình vuông cắt nhau tại điểm thứ hai N.
a) Chứng minh rằng đường thẳng AN đi qua một đỉnh của hình vuông thứ hai
b) Tìm quỹ tích của điểm N khi điểm M di động trên AB
c) Tìm quỹ tích trung điểm I của đoạn thẳng nối hai tâm hình vuông
HD a) 0 0 0
45 45 90ANB ANM NMB     , tương tự: 0
90DNB  ; b) N nằm trên đường tròn
đường kính AB; giới hạn trong nửa đường tròn; c) Từ hai tâm hạ hai đường vuông góc; khoảng
cách từ I đến AB bằng AB/4, giới hạn khi M trùng A, M trùng B là đoạn PQ (P, Q cũng chính là
giao của đt cách AB một đoạn AB/4 cắt AE và BD).
Luyện tập Góc …
Bài 4. Cho tam giác ABC nội tiếp đường tròn. Lấy điểm D trên cung BC (không chứa A) của
đường tròn đó. Vẽ DH vuông góc với BC; DI vuông góc với CA và BK vuông góc với AB.
Chứng minh rằng:
BC AC AB
DH DI DK
 
M
P
Q
A
B
Luyện thi học sinh giỏi, thi chuyên toán lớp 10 Thầy Hồng Trí Quang
3
HD ý tưởng của bài toán là tìm điểm M trên BC để tách tỉ số:
BC BM MC
DH DH DH
  , và từng tỉ số
của tổng này ứng với tỉ số cần chứng minh.
Trên cạnh BC lấy điểm M sao cho BMD ADC
( . )BMD ADC g g 
DH BM
DI AC
  (tỉ số đường cao bằng tỉ số đồng dạng)
AC BM
DI DH
 
Chứng minh tương tự thì:
AB CM
DK DH

Cộng lại ta có Đpcm
Bài 5. Tam giác ABC vuông tại A, nội tiếp đường tròn (O) đường kính 4 2cm. Tiếp tuyến tại
C của đường tròn cắt tia phân giác góc B tại K. Tính độ dài BK biết BK cắt AC tại D và BD =
4cm.
Hướng dẫn
Gọi giao điểm hai đường tròn là M. Đặt MD = MK = x. Từ 2
.BC BM BK ; 2 17 2BK  
(cm)
Bài 6. Từ điểm A nằm ngoài đường tròn (O) kẻ hai tiếp tuyến AB, AC (B, C là tiếp điểm). Gọi
M là trung điểm AC. Đoạn thẳng MB cắt đường tròn tại K (khác B). Tia AK cắt đường tròn tại
D (khác K). Chứng minh rằng BD song song với AC
HD
Ta có: MKC CBM MCK MBC  
MC MK
MB MC
 
MC MK
MB MA
  (vì MA = MC)
Vì AMK chung nên MAK MBA  MAK ABM 
Mà BDK ABM nên BDK KAM Đpcm
Bài 7. Cho hai đường tròn (O; R) và (O’, r) tiếp xúc trong tại A (R > r). Dây BC của đường tròn
(O;R) tiếp xúc với (O’; r) tại M. Chứng minh rằng AM là tia phân giác góc BAC
HD Kẻ tiếp tuyến chung từ A cắt BC tại F.
Vì: MBA CAF , mà FMA FAM suy ra Đpcm
Bài 8. Cho hai đường tròn (O) và (O’) tiếp xúc ngoài tại I, AB là tiếp tuyến chung. Kẻ đường
kính AOD, từ D kẻ tiếp tuyến DE với đường tròn (O’). Chứng minh tam giác DAE cân.
Luyện thi học sinh giỏi, thi chuyên toán lớp 10 Thầy Hồng Trí Quang
4
HD a) Kẻ tiếp tuyến chung tại I; b) Kẻ tiếp tuyến chung AB, chứng minh D, I, B thẳng hàng;
 2 2
.DB DDE DI A
Bài 9. Từ điểm M nằm ngoài đường tròn (O), vẽ hai tiếp tuyến MA, MB và cát tuyến MCD với
đường tròn (O). Gọi I là giao điểm của AB và CD. Chứng minh rằng:
IC MC
ID MD

HD: Khi chứng minh các đẳng thức tích, người ta thường vận dụng các tam giác đồng dạng.
Nhưng nếu không ghép được trực tiếp vào các tam giác đồng dạng thì nên vận dụng thông qua
các đẳng thức trung gian.
Từ các cặp tam giác đồng dạng ta có:
( . )IAC IDB g g 
AC IC
DB IB
 
( . )IBC IDA g g 
BC IB
AD ID
 
Suy ra: . .
IC IC IB AC BC
ID IB ID DB DA
 
Từ đó ta chứng minh: .
MC AC BC
MD DB DA

Xét các cặp tam giác đồng dạng
( . )MAC MDA g g 
AM MC AC
DM MA DA
  
AM AC
DM DA
  và 2
.MA MC MD
( . )MBC MDB g g 
BM BC
DM DB
 
Ta có: 2
.MC MC MD
MD MD

2
2
MA
MD
 .
MA MA
MD MD

AC BC
AD BD

Bài 10.*Cho hai đường tròn (O) và (O’) cắt nhau tại A và B. Các điểm M, N theo thứ tự di
chuyển trên (O), (O’) sao cho chiều đi từ A đến M và từ A đến N trên các đường tròn theo chiều
kim đồng hồ và sđ AM = sđ AN . Chứng minh rằng đường trung trực MN luôn đi qua một điểm
cố định.
Hướng dẫn
Vẽ hình dự đoán điểm cố định nằm trên đường thẳng đi qua A, vuông góc AB
Qua A kẻ đường thẳng vuông góc với AB cắt (O) và (O’) lần lượt tại C, D. Gọi I là trung
điểm CD thì I cố định. Ta chứng minh I nằm trên trung trực MN.
Luyện thi học sinh giỏi, thi chuyên toán lớp 10 Thầy Hồng Trí Quang
5
Gọi H là trung điểm MN thì IH là đường trung bình của hình thang vuông MDNC.
Đpcm.
Bài 11.*Cho hai đường tròn (O; R) và (O’, R’) cắt nhau tại hai điểm A, B. Tiếp tuyến với (O) tại
A cắt (O’) tại C. Tiếp tuyến với (O’) tại A cắt (O) tại D. Gọi M là giao điểm hai đường thẳng
AB và CD. Gọi N là trung điểm của CD. Chứng minh rằng:
1/
2
2
'MC R
MD R
 2/ CAM DAN
Hướng dẫn
1/ BAC BDA 
'BA BC AC R
BD BA DA R
    và BM là phân giác góc DBC
Từ đó:
2
2
'MC BC BC BA R
MD BD BA BD R
  
2/ Gọi I là trung điểm AD, cộng trừ góc ta có: AIN ABC . Từ đó:
( . . )AIN ABC c g c  . Đpcm
Bài 12. *Cho đường tròn (O), từ điểm M nằm ngoài đường tròn kẻ hai tiếp tuyến MA và MB.
Đường tròn đi qua M và tiếp xúc với AB tại B cắt đường tròn (O) tại C. Chứng minh rằng AC đi
qua trung điểm MB.
Dự đoán h.b.h, tạo hình bình hành
HD Gọi D là giao điểm của AC với đường tròn qua M tiếp xúc AB. ABDM là h.b.h do
DM C CAB
Bài 13.*Cho tam giác ABC nội tiếp đường tròn (O), A là điểm cố định; B và C thay đổi. Kẻ
đường cao BH và CK. Chứng minh rằng HK luôn song song với một đường thẳng cố định.
HD Kẻ tiếp tuyến Ax, chứng minh AKH KAx . Chú ý: chứng minh AKH HCB vì cùng phụ
với góc HBC
Bài 14. *Cho đường tròn (O), từ điểm M nằm ngoài đường tròn kẻ hai tiếp tuyến MA,
MB, gọi D là điểm trên cung lớn AB , đường thẳng AD cắt MB kéo dài tại E sao cho D là
trung điểm của AE. Đường thẳng MD cắt đường tròn (O) tại I. Chứng minh rằng BI song
song với AD.
GÓC TRONG VÀ GÓC NGOÀI ĐƯỜNG TRÒN
Chú ý: Hai dây cung song song tạo ra hai cung có số đo bằng nhau.
Luyện thi học sinh giỏi, thi chuyên toán lớp 10 Thầy Hồng Trí Quang
6
Bài 15. Cho đường tròn (O; R) với ba dây liên tiếp AB, BC, CD bằng nhau và cùng nhỏ hơn R.
Các đường thẳng AB, CD cắt nhau tại I, các tiếp tuyến của đường tròn tại B và D cắt nhau tại K.
Chứng minh rằng:
1/ BIC BKD 2/ IBC KBD  ; CBD IBK 
Hướng dẫn 2/ TH1 (g.g), đồng dạng thứ 2 (c.g.c)
HD cộng trừ góc – bài dễ
Bài 16.Cho tam giác ABC (AC < AB) nội tiếp trong đường tròn tâm O bán kính R. Đường phân
giác trong và ngoài của góc A cắt đường thẳng BC theo thứ tự tại D, E sao cho AD = AE. Tính
2 2
AB AC theo R.
Hướng dẫn:
Đưa hai cạnh AB, AC về độ dài hai cạnh của tam giác vuông.
Gọi AD cắt (O) tại M, kẻ đường kính BF. Vì: FA AC nên AF = AC. Do đó:
2 2 2
4AB AC R 
Bài 17. Cho tam giác ABC có AD là đường phân giác. Vẽ đường tròn tâm (O) đi qua A và
D đồng thời tiếp xúc với BC tại D. Đường tròn này cắt AB, AC lần lượt tại E và F. Chứng
minh rằng:
a) EF song song với BC
b) AED ADC  và AFD ADB 
c) 2
. .AE AC AF AB AD 
Bài 18. Cho đường tròn (O; R) với ba dây cung liên tiếp AB, BC, CD bằng nhau và cùng
nhỏ hơn R. Các đường thẳng AB, CD cắt nhau tại I. Các tiếp tuyến của đường tròn tại B và
D cắt nhau tại K. Chứng minh rằng
a) BIC BKD
b) IBC KBD 
c) CBD IBK 
Bài 19.Từ điểm E ở bên ngoài đường tròn (O) kẻ hai cát tuyến EAB, EDC sao cho AB <
CD. Tia DA và CB cắt nhau tại F. Tia phân giác của hai góc CEB và CFD cắt nhau tại I.
Chứng minh rằng EI FI
Hướng dẫn Để chứng minh 0
90FIE  ta chứng minh: PN MQ NQ MP  
PP CHỨNG MINH TỨ GIÁC NỘI TIẾP VÀ ỨNG DỤNG
Luyện thi học sinh giỏi, thi chuyên toán lớp 10 Thầy Hồng Trí Quang
7
Dấu hiệu 1 Chỉ ra hai đỉnh kề nhau cùng nhìn một cung một góc bằng nhau (ví dụ D DCA CB )
Dấu hiệu 2: Chỉ ra tổng hai góc đối nhau bằng 1800
.
Dấu hiệu 3: Mở rộng
Nếu tứ giác ABCD có AB cắt CD tại M thỏa mãn MA.MB = MC.MD thì ABCD nội tiếp
Nếu AC cắt BD tại N thỏa mãn NA.NC = NB.ND thì ABCD nội tiếp.
Chú ý.
1. Điểm M nằm ngoài đường tròn (O). Nếu qua M kẻ tiếp tuyến MN và hai cát tuyến MAB,
MCD thì 2
. .MAMB MC MD MN 
2. Chứng minh năm điểm A, B, C, D, E thuộc cùng một đường tròn,
+) Ta chứng minh 2 tứ giác bất kì (ví dụ ABCD, ABCE) nội tiếp.
+) Ta chứng minh 5 điểm cách đều một điểm cho trước
Dấu hiệu 1. Tổng hai góc đối bằng 1800
Bài 20. Từ 1 điểm M ở ngoài (O), vẽ 2 tiếp tuyến MA, MB với đường tròn. Trên cung nhỏ AB
lấy 1 điểm C. Vẽ CD vuông góc với AB, CE vuông góc với MA, CF vuông góc với MB. Gọi I là
giao điểm của AC và DE, K là giao điểm của BC và DF. Chứng minh rằng
a) Tứ giác ICKD nội tiếp
b) IK vuông góc với CD
c) CD2
= CE.CF (góc tiếp tuyến và dây cung – hai tam giác đồng dạng)
a) ta có: 1 1A B (cùng chắn cung AC)
+ do tứ giác BFCD nt 1 1F B (cùng chắn cung CD)
C
2
2
2
2
2
1
1
1
1
1
K
I
F
E
D
O
B
M
A
Luyện thi học sinh giỏi, thi chuyên toán lớp 10 Thầy Hồng Trí Quang
8
Suy ra: 1 1F A (1)
+ do tứ giác AECD nt 1 1A D (cùng chắn cung CE) (2)
Từ (1) và (2) suy ra: 1 1 1F D B 
Mặt khác: 2 2A B (cùng chắn cung BC)
+ do tứ giác AECD nt 2 2A E (cùng chắn cung CD)
Suy ra: 2 2E B (3)
+ do tứ giác BFCD nt 2 2D B (cùng chắn cung CF) (4)
Từ (3) và (4) suy ra: 2 2 2E D A 
b) Xét tứ giác ICKD, ta có: 0
1 2 1 2 180ICK IDK ICK D D ACB B A        (tổng các góc
của tam giác ABC), mà ;ICK IDK là 2 góc ở vị trí đối nhau, suy ra tứ giác ICKD nt
d) ta có tứ giác ICKD nt 1 2I D (cùng chắn cung CK), mà 2 2D A (cmt)
Suy ra 1 2I A , mà 1 2;I A là 2 góc ở vị trí đồng vị nên IK // AB, lại do AB vuông góc với CD,
nên IK vuông góc với CD
c) Xét tam giác CDE và tam giác CDF, ta có:
 1 1 2
2 2
. .
D F CD CE
CDE CFD g g CD CE CF
CF CDE D
 
      
 
Bài 21.Cho đường tròn tâm O đường kính AB. Trên một nửa đường tròn đường kính AB lấy
điểm C, D sao cho AC AD (D khác B). Trên nửa đường tròn còn lại lấy điểm E (khác A và
B). CE cắt AD tại I. Đường thẳng IO cắt BE tại K.
a) Gọi F là điểm đối xứng của D qua IK, chứng minh tứ giác IFEK nội tiếp
b) Chứng minh tam giác CDK vuông.
Hướng dẫn. Bỏ a
a) BIK DIK FDB FAB  
Mà 0
180FAB FEB  nên tứ giác IFEK nội tiếp.
Luyện thi học sinh giỏi, thi chuyên toán lớp 10 Thầy Hồng Trí Quang
9
b) Ta có IEK IFK IDK 
Vậy 0
180IDK CDB  0 0
90 180IDK CDI    . Đpcm.
Tự luyện
Bài 22.Cho tam giác nhọn ABC (AB < AC). Các đường cao BD, CE cắt nhau tại H. Gọi M là
giao điểm của BC và DE. Gọi O là trung điểm cạnh BC. Gọi P (khác O) là giao điểm của các
đường tròn ngoại tiếp các tam giác OBE và OCD. Chứng minh rằng:
a) Tứ giác HEPD nội tiếp
b) Tứ giác MEPC nội tiếp
c) Chứng minh MP AO
Hướng dẫn a; b bỏ
a) Chứng minh A, D, P, E, H cùng nằm trên đường tròn.
b) Chú ý. O là tâm ngoại tiếp BEDC, và sử dụng a chứng minh b.
   DEC ODB PED PDO PCO
c) MPC MEC và   MEB OCD ODC OPC nên MP PO
Tuy nhiên, cần chứng minh A, P, O thẳng hàng.
Do 0
180   APD DPO OCD DPO
Dấu hiệu 2. Hai góc cùng nhìn một cung
Bài 23.Cho hai đường tròn (O) và (O’) cắt nhau tại A và B. Đường thẳng d cắt (O) và (O’) lần
lượt tại C và D. Hai tia CO và DO’ cắt nhau ở E. Chứng minh rằng C, A, D, E cùng thuộc một
đường tròn.
HD 'D  AOC AO ACO ADE
Bài 24.Cho tam giác ABC, điểm M di động trên cạnh BC. Các đường trung trực của các đoạn
thẳng BM, CM lần lượt cắt AB, AC tại D, E.
a) Gọi S là điểm đối xứng của M qua DE, SM cắt AH tại K, chứng minh tứ giác SAKB nội tiếp
b) Chứng minh rằng đường thẳng qua M và vuông góc với DE luôn đi qua điểm cố định.
Hướng dẫn. Bỏ a
a) Chứng minh
1
2
BSK BDM ABK 
Luyện thi học sinh giỏi, thi chuyên toán lớp 10 Thầy Hồng Trí Quang
10
b) Đã có BSAK nội tiếp, nhưng chưa đủ suy ra K cố định. Ta cần chứng minh S, K thuộc đường
tròn ngoại tiếp tam giác ABC, tức chứng minh SACK nội tiếp (chứng minh tương tự).
Tự luyện
Bài 25.Cho tam giác ABC nội tiếp (O) có 2AB < BC. Trên cạnh BC lấy điểm D sao cho CD =
AB. Đường thẳng qua D song song với AC cắt AB ở E. Tia phân giác góc ABC cắt DE ở F.
a) Gọi N là giao điểm của AF và đường tròn (O. Đường thẳng qua D song song với AB cắt AF ở
M. Chứng minh tứ giác DNMC nội tiếp.
b) Chứng minh rằng đường thẳng AF đi qua trung điểm I của BD.
Hướng dẫn. Bỏ a
a) Ta có NMD BAN NCD 
b) Để chứng minh I là trung điểm BD ta chứng minh tứ giác ADMB là hình bình hành, tức
chứng minh hai cặp cạnh đối AB và DM song song và bằng nhau.
2 2 2BDM ABD FBD FND DCM   
Mà BDM ABD DCM DMC   nên DM = DC = AB.
Bài 26.Cho đường tròn (O) đường kính AB. Trên tiếp tuyến của đường tròn (O) tại A lấy điểm
M, vẽ cát tuyến MCD (C nằm giữa M và D). Gọi E, F lần lượt là giao điểm của OM với BC và
BD.
a) Kẻ tiếp tuyến MN, chứng minh tứ giác MCNE nội tiếp.
b) Chứng minh rằng OE = OF.
Hướng dẫn. Bỏ a
a) Chứng minh NME NAO NCE 
b) AOE BOF   EAO OBF  MAE OAD  MNE BCD  (Theo a)
Cách 2. Qua C kẻ song song với EF cắt AB, BD tại H, K. Kẻ ON vuông CD tại N, chứng minh
ACHN nội tiếp và NH song song BD. Suy ra H là trung điểm CK. Sử dụng talet ta chứng minh
được
OE BO OF
CH BH HK
 
Bài 27.*Cho tam giác nhọn ABC. Trên nửa mặt phẳng bờ AB không chứa C vẽ tia Ax, trên nửa
mặt phẳng bờ AC không chứa B vẽ tia Ay sao cho 0
90yAC xAB  . Vẽ BD Ax tại D,
CE Ay tại E. Vẽ AH là đường cao của tam giác ABC. Gọi M là trung điểm BC. Chứng minh
rằng M, H, D, E cùng thuộc một đường tròn.
Hướng dẫn
Chứng minh hai góc cùng nhìn cung bằng nhau
Luyện thi học sinh giỏi, thi chuyên toán lớp 10 Thầy Hồng Trí Quang
11
Ta có 0
180DHE DHB EHC   0
180 2DAB 
Để sử dụng giả thiết M là trung điểm BC, ta lấy thêm I, K lần lượt là trung điểm AB, AC.
Khi đó ( . . )AIM MKE c g c  
DMH AMI IMK KME    IDM DMI BAC  
 0 0
180 180DIM BAC DIB     0
180 2DAB  DHE
Tam giác bằng nhau, tam giác đồng dạng.
Bài 28.Cho nửa đường tròn tâm O, đường kính AB, M là điểm đối xứng của O qua A. Đường
thẳng qua M cắt nửa đường tròn (O) tại C và D (C nằm giữa M và D). Gọi E là giao điểm của
AD và BC.
a) Gọi N là trung điểm của OA, chứng minh tứ giác NEDB nội tiếp
b) Chứng minh rằng 3
BC AE
AD BE

HD Bỏ a.
a) Ta chứng minh NDE NBE
Có NBE ADC
Nên NDE NBE NDA ADC  DMA NDO  DMO NDO   (c.g.c)
b) Áp dụng tính chất của tứ giác nội tiếp ta có
Bài 29.*Cho tam giác ABC vuông tại A có AM là đường trung tuyến, AH là đường cao. Trên tia
đối của tia AM lấy điểm P (P khác A). Các đường thẳng qua H vuông góc với AB và AC lần
lượt cắt các đường thẳng PB và PC tại Q và R tương ứng.
a) Gọi E là giao điểm của AC và PB, F là giao điểm của AB và PC. Qua P kẻ đường thẳng song
song với BC cắt AC, AB lần lượt tại E’, F’. Gọi I là giao điểm của HQ và AB, K là giao điểm
của HR và AC. Chứng minh IK song song với QR.
b) Chứng minh tứ giác BHAS nội tiếp với S là giao điểm của RA và PB
c) Chứng minh A là trực tâm tam giác PQR
Hướng dẫn. Bỏ a; b
3 . 3 . . .  AE AE AN AB BN BA BE BC
Luyện thi học sinh giỏi, thi chuyên toán lớp 10 Thầy Hồng Trí Quang
12
a) Talet, chứng minh P là trung điểm E’F’
Talet, chứng minh EF song song BC.
Talet, chứng minh IK song song với QR
b) ;  
HQ HI HB
BHQ AHR
HR HK HA
(c. .c)  BHQ AHE g nên BHAS nội tiếp
c) SR vuông PQ, PA vuông QR nên A là trực tâm.
Tự luyện
Bài 30. Nếu hai đường thẳng AB, CD cắt nhau tại điểm M và . .MAMB MC MDthì bốn điểm
A, B, C, D thuộc cùng một đường tròn.
Bài 31.Cho đường tròn (O), điểm K nằm ngoài đường tròn. Kẻ các tiếp tuyến KA, KB và cát
tuyến KCD với đường tròn. Gọi M là giao điểm của OK và AB. Kẻ OH vuông góc với CD, cắt
AB ở E. Chứng minh rằng:
a) CMOE là tứ giác nội tiếp
b) CE, DE là các tiếp tuyến của (O).
HD cm góc CME = COE
Bài 32. Cho tam giác ABC có 3 góc nhọn. Đường tròn tâm O đường kính BC cắt AB tại E, cắt
AC tại F. Các tia BE cà CE cắt nhau tại H. Gọi M là trung điểm của BH. Chứng minh rằng tứ giác
EMKF
Bài 33. Cho tam giác ABC. Các điểm D, E di động trên các tia BA, CA sao cho 3BD = 2CE.
a) Vẽ (O’) ngoại tiếp tam giác ABC. Gọi M là điểm chính giữa cung BC (M và A nằm khác phía
đối với BC). I là điểm trên cạnh BC và 3 2BI IC , MI cắt (O’) tại N khác M.
a) Chứng minh tứ giác ADEN nội tiếp.
b) Chứng minh rằng tâm O của đường tròn ngoại tiếp tam giác ADE thuộc một đường thẳng cố
định.
Hướng dẫn. Bỏ a
a)
NB IB BD
NC IC CE
  nên NBD NCE  . Đpcm
b) Tứ giác ADEN nội tiếp nên OA = ON, tức O thuộc đường thẳng cố định là trung trực của AN
Bài 34.*Cho tam giác ABC nhọn, ba đường cao AD, BE và CF cắt nhau tại H. Qua A vẽ các
đường thẳng song song với BE, CF lần lượt cắt các đường thẳng CF, BE tại P và Q.
a) Chứng minh tứ giác AKEQ nội tiếp.
b) Chứng minh rằng PQ vuông góc với trung tuyến AM của tam giác ABC.
Hướng dẫn. Bỏ a
Luyện thi học sinh giỏi, thi chuyên toán lớp 10 Thầy Hồng Trí Quang
13
a) Gọi I là giao điểm của AH và PQ, K là giao điểm của AM và PQ. Ta có I là trung điểm AH.
MAC IQH AMC QIH   ;
AC MC
ACM IHQ
QH IH
  
ACM IHQ (Hai góc nhọn có cạnh tương ứng vuông góc)
AC MC
QH IH

AC BC
QH AH
  ( . )ABC QAH g g   (hai góc có cạnh tương ứng vuông
góc)
b) Hiển nhiên
Bài 35.*Cho hình thoi ABCD có 0
60BAD  . Đường thẳng d qua C cắt AB, AD lần lượt ở M và
N. Gọi K là giao điểm của BN và DM. Chứng minh rằng A, B, D, K cùng thuộc một đường tròn.
Hướng dẫn
Từ A kẻ đường thẳng song song với BD, E thuộc BN
MA NA AE
CD ND BD
  MA AE  ( . . )AEB AMD c g c    . Đpcm.
DẤU HIỆU 4. SỬ DỤNG DẤU HIỆU TÍCH ĐỂ CHỨNG MINH TỨ GIÁC NỘI TIẾP
Trong chương trình toán nâng cao hình học lớp 9, chứng minh tứ giác và sử dụng tứ giác nội
tiếp là rất quan trọng. Trong đó dấu hiệu tích là dấu hiệu để nhận biết và chứng minh nhanh tứ
giác nội tiếp mà các bạn chưa khai thác. Một vài ví dụ dưới đây sẽ chỉ cho các bạn thấy ứng
dụng đơn giản mà tuyệt vời này.
Tính chất tích
Thuận. Nếu hai đường thẳng chứa các dây AB, CD của một đường tròn cắt nhau tại một điểm M
(nằm trong hoặc nằm ngoài đường tròn) thì: . .MAMB MC MD
Đảo. Nếu hai đường thẳng AB, CD cắt nhau tại điểm M và . .MAMB MC MDthì bốn điểm A,
B, C, D thuộc cùng một đường tròn.
Hướng dẫn. Sử dụng tam giác đồng dạng để chứng minh.
Chú ý. Nếu M nằm ngoài đường tròn (O), cát tuyến MAB và tiếp tuyến MP thì 2
.MAMB MP
Bài 36.Từ điểm A nằm ngoài đường tròn (O) vẽ các tiếp tuyến AM, AN đến đường tròn (O) và
cát tuyến ADC. B là điểm trên cung CM (không chứa D). Gọi H là giao điểm của MN và BD, E
là giao điểm của CH và đường tròn (O).
Luyện thi học sinh giỏi, thi chuyên toán lớp 10 Thầy Hồng Trí Quang
14
a) Gọi K là giao điểm của AH và đường tròn ngoại tiếp tam giác (AMN). Chứng minh tứ giác
AEKC và HKCD nội tiếp.
b) Chứng minh rằng A, E, B thẳng hàng.
Hướng dẫn. Bỏ a
a) Xét . . .HKHE HC HM HN HA 
2
. . .AH AK AF AO AN AC AD  
b) 0
180AEC BEC HKC EDC    . Đpcm
Bài 37.Cho điểm A nằm ngoài đường tròn (O). Vẽ AB, AC là các tiếp tuyến và cát tuyến ADE
(tia AD nằm giữa hai tia AB và AO). OA cắt BC tại H. Vẽ BK vuông góc với DE tại K, KH cắt
AB tại G và cắt đường thẳng đi qua A song song với CD tại M. Vẽ AS vuông góc với GD tại S.
Chứng minh tứ giác DKMS nội tiếp
Hướng dẫn
Hạ DT vuông góc với AB tại T
Ta có . .GT GA GDGS
Nên tứ giác DKMS nội tiếp . .GT GA GK GM   tứ giác ATKM nội tiếp BTK AMK 
Ta có BTK TKA TAK VCH VHC DVH AMK     
Bài 38. Cho hình thang ABCD (AB//CD, AB < CD) có đường tròn (O) qua A, D tiếp xúc với
BC ở N, đường tròn (I) đi qua B, C tiếp xúc với AD ở M. Chứng minh rằng tứ giác MNCD nội
tiếp
Hướng dẫn
Gọi S là giao điểm của AD và BC
Xét tích ta có 2
.SM SB SC ; 2
.SN SASD
Cần chứng minh . .SM SD SN SC
2 2
2 2
SM SC
SN SD
 
2
2
.
.
SB SC SC
SA SD SD
 
SB SC
SA SD
  (luôn đúng)
I
H
O
G
A
B
F
N
K
H
E
I
N
C
Luyện thi học sinh giỏi, thi chuyên toán lớp 10 Thầy Hồng Trí Quang
15
Bài 39.Cho đường tròn (O) có dây cung BC (khác đường kính) cố định, A là điểm chuyển động
trên cung lớn BC, M là trung điểm dây BC. Gọi D là giao điểm của AM và cung nhỏ BC, N là
giao điểm của AB và CD.
a) Gọi E là giao điểm của các tiếp tuyến của đường tròn (O) tại B và C. Chứng minh tứ giác
AODE nội tiếp, tứ giác BNED nội tiếp.
b) Chứng minh rằng N thuộc một đường thẳng cố định.
Hướng dẫn. Bỏ a
a) Dễ thấy MA.MD = MB.MC = MD.MO
Để chứng minh tứ giác NEDB nội tiếp, ta chứng minh hai góc bằng nhau
NDE NBE EDC EBA  EKB EBA  AEO DEO  OA OD  (luôn đúng)
b) END NBD END BCD  / /EN BC
Vậy N thuộc đường thẳng cố định, đi qua E cố định và song song với BC.
Bài 40. Cho tứ giác BFEC nội tiếp đường tròn đường kính BC (BF < CE). BE cắt CF tại H, FD
cắt đường tròn (O) tại M, ED cắt đường tròn (I) tại N. Chứng minh tứ giác DMSN nội tiếp
Hướng dẫn
Gọi A là giao điểm BF và CE, ta có A, H, D thẳng hàng
; . . .FD FM FH FC FAFB  ; . . .EEN ED EH EB EA C 
. : . .
FM FA FB FD FA FB ED
EN EA EC ED EA FD EC
  . . 1
AC BC AB
AB AC BC
  FM EN 
Vậy (c.g.c)  SFM SEN . Đpcm.
Bài 41.*(Thi hsg cấp tỉnh – đề số 3) Cho hai đường tròn (O1) và (O2) tiếp xúc ngoài nhau tại
điểm T. Hai đường tròn này nằm trong đường tròn (O3) và tiếp xúc với (O3) tương ứng tại M và
N. Tiếp tuyến chung tại T của (O1) và (O2) cắt (O3) tại P. PM cắt đường tròn (O1) tại điểm thứ
hai A và MN cắt (O1) tại điểm thứ hai B. PN cắt đường tròn (O2) tại điểm D và MN cắt (O2) tại
điểm thứ hai C.
a) Chứng minh rằng tứ giác AMND là tứ giác nội tiếp
b) Chứng minh AB song song với PN.
c) Gọi E là giao điểm của AB và CD, chứng minh . .EB PN EC PM
b) Chứng minh rằng tứ giác ABCD nội tiếp.
Luyện thi học sinh giỏi, thi chuyên toán lớp 10 Thầy Hồng Trí Quang
16
HD
a) Theo dấu hiệu ta có
2
. .PA PM PT PD PN  nên tứ giác AMDN nội tiếp
b) 1 1
3 3
O A O BMA MB
MP O P O N MN
   / /AB NP
c) EBC PNM 
d)
EB PM PA EA
EC PN PD ED
  
Luyện tập
Bài 42.Cho tam giác nhọn ABC. Vẽ đường tròn (O) đường kính BC. Vẽ AM là tiếp tuyến của
(O). Kẻ MK vuông góc với OA tại K. Gọi H là trực tâm của tam giác ABC, D là giao điểm của
AH và BC. Chứng minh rằng D, H, K, O cùng thuộc một đường tròn.
Hướng dẫn. Xét tích 2
. . .AH AD AN AB AM AK AO  
Bài 43.Cho đường tròn (O), điểm K nằm ngoài đường tròn. Kẻ các tiếp tuyến KA, KB và cát
tuyến KCD với đường tròn. Gọi M là giao điểm của OK và AB. Chứng minh rằng:
a) CMOD là tứ giác nội tiếp
b) Đường thẳng AB chứa tia phân giác của góc CMD.
Hướng dẫn
C M O
K
A
B D

E
N
M
B
C
O1
O3
O2
D
P
A
T
Luyện thi học sinh giỏi, thi chuyên toán lớp 10 Thầy Hồng Trí Quang
17
a) 2
. . MAMO MA MC MD nên tứ giác OMCD nội tiếp
b) Để chứng minh CMB DMB ta chứng minh CMK DMO
Thật vậy   CMK ODC OCD DMO
Bài 44.Cho tam giác ABC, đường cao AH. Vẽ đường tròn tâm H bán kính HA. D là điểm nằm
trên đường tròn (H). Gọi M, N lần lượt là trung điểm của DB và DC. Chứng minh rằng bốn
điểm D, M, H, N cùng thuộc một đường tròn.
Hướng dẫn
Gọi E là giao điểm của DH và (H) thì 2
. .BH HC AH HD HE  nên DBEC nội tiếp.
BED BCD  MHD MND 
Bài 45. Hsg TP HN. Cho tam giác ABC có ba góc nhọn, nội tiếp đường tròn (O) và AB < AC.
Các đường cao AD, BE, CF của tam giác ABC cắt nhau tại H. Gọi I là giao điểm của hai đường
thẳng EF và CB. Đường thẳng AI cắt (O) tại M (M khác A).
a) Chứng minh năm điểm A, M, F, H, E cùng nằm trên đường tròn.
b) Gọi N là trung điểm BC, chứng minh ba điểm M, H, N thẳng hàng.
c) Chứng minh BM.AC + AM.BC = AB.MC
Hướng dẫn
a) Dễ dàng nhận thấy tứ giác AFHE nội tiếp,
tức A, F, H, E đã nằm trên 1 đường tròn.
Ta chứng minh tứ giác AMFH,
AMFE hoặc MEHF nội tiếp sẽ có đpcm.
Ta chứng minh AMFE nội tiếp.
Thật vậy, theo dấu hiệu tích . . . IM IA IB IC IF IE Đpcm.
b) Ta chứng minh HN, HM cùng vuông góc với AI, khi đó M, H, N thẳng hàng. Thật vậy
0 0
180 90HMA HAE  
Sử dụng bài toán cơ bản. Cho tam giác ABC nội tiếp đường tròn (O) có trực tâm H, đường kính
AD. Khi đó DH cắt BC tại trung điểm mỗi đường
Áp dụng. Nếu HN kéo dài cắt (O) tại D thì A, O, D thẳng hàng. Khi đó NH vuông góc với IA
A
B
C
F
E
I
M
H
N
Luyện thi học sinh giỏi, thi chuyên toán lớp 10 Thầy Hồng Trí Quang
18
Vậy HM, HN cùng vuông góc với IA nên H, M, N thẳng hàng
c) Sử dụng định lí P tô lê mê (xem chi tiết về Định lí P tô lê mê trong chuyên đề tứ giác nội tiếp)
Bài 46.Chuyên toán ĐHSPHN 2008 - 2009 Cho tam giác ABC vuông tại C. Trên cạnh AB lấy
điểm M tùy ý (M khác A và B). Kí hiệu 1 2; ;O O O lần lượt là tâm của các đường tròn ngoại tiếp
các tam giác ABC, AMC và BMC.
a) Chứng minh bốn điểm 1 2; ; ;C O M O cùng nằm trên một đường tròn ( )
b) Chứng minh O cũng nằm trên đường tròn ( )
c) Xác định vị trí điểm M để đường tròn ( ) có bán kính nhỏ nhất
HD c) Gọi R là bán kính của (C) thì 𝐶𝑂 ≤ 2𝑅. Khi đó M là hình chiếu của C trên AB
Bài 47.(Chuyên KHTN 2002 – 2003) Cho hình vuông ABCD, M là điểm thay đổi trên cạnh BC
(M không trùng với B), N là điểm thay đổi trên cạnh CD (N không trùng với D) sao cho:
MAN MAB DAN 
a) BD cắt AN và AM tương ứng tại P và Q. Chứng minh rằng năm điểm P, Q, M, C, N cùng
nằm trên một đường tròn.
b) Chứng minh rằng đường thẳng MN luôn tiếp xúc với một đường tròn cố định khi M, N thay
đổi
c) Kí hiệu 1S là diện tích tam giác APQ, 2S là diện tích tứ giác PQMN. Chứng minh rằng tỉ số:
1
2
S
S
không đổi khi M và N thay đổi.
d) Tìm GTNN của MN
HD ADNQ nội tiếp; các điểm cùng thuộc đường tròn đk MN; b) (A; AD); Tỉ số diện tích bằng
bình phương đồng dạng; đs 1d) Tính    2 2 2
2 2 2MN CN CM CN CM   
Bài 48.Cho tam giác nhọn ABC (AB < AC), hai đường cao BD và CE cắt nhau tại H. Gọi I là
trung điểm của BC, đường tròn qua B, E, I và đường tròn đi qua C, D, I cắt nhau tại K (K khác
I)
a) Chứng minh năm điểm A, E, H, K, D nằm trên cùng một đường tròn
b) *Đường thẳng DE cắt BC tại M. Chứng minh ba điểm M, H, K thẳng hàng.
Luyện thi học sinh giỏi, thi chuyên toán lớp 10 Thầy Hồng Trí Quang
19
HD a) AEKD nt, AEHD nt; b) A, K, I thẳng hàng; ICK IAC ; MEKC nội tiếp;
0
90MKI BEC 
Bài 49.Cho tam giác ABC không cân, nội tiếp đường tròn (O). Gọi CD là đường kính của đường
tròn, qua D kẻ tiếp tuyến với đường tròn cắt đường thẳng AB tại E, EO cắt cạnh BC, CA tại M
và N tương ứng. Gọi I là trung điểm AB. Chứng minh rằng
a) Bốn điểm O, D, E, I nằm trên một đường tròn b) *O là trung điểm MN
HD a) Đường tròn đk OE; b) Từ A kẻ đường thẳng song song OE cắt BC tại F; tứ giác AIDJ nt;
IJ song song BC nên JA = JF, đpcm.
Bài 50. Cho tam giác nhọn ABC (AB < AC), hai đường cao BD và CE cắt nhau tại H (D
trên AC, E trên AB). Gọi I là trung điểm BC, đường tròn qua B, E, I và đường tròn qua C, D,
I cắt nhau tại K (K khác I).
a) Chứng minh rằng năm điểm A, E, H, K, D nằm trên một đường tròn và 𝐵𝐷𝐾̂ = 𝐶𝐸𝐾̂
b) Đường thẳng DE cắt BC tại M. Chứng minh ba điểm M, H, K thẳng hàng.
HD chứng minh 2 tứ giác AEKD, AEHD nội tiếp
b) 5 điểm A, E, H, K, D nằm trên đường tròn đường kính AH nên HK AI (1)
0
180AKE EKI  nên A, K, I thẳng hàng.
ICK DEK nên MEKC nội tiếp MEC MKC 
Mặt khác MEB AED 0
90MKI BEC   MK AI  (2)
Từ (1) và (2) thì M, H, K thẳng hàng
Bài 51.Cho nửa đường tròn tâm O, đường kính AB, M là điểm đối xứng của O qua A. Đường
thẳng qua M cắt nửa đường tròn (O) tại C và D (C nằm giữa M và D). Gọi E là giao điểm của
AD và BC. Chứng minh rằng 3
BC AE
AD BE

Bài 52.Cho tam giác ABC. Các điểm D, E di động trên các tia BA, CA sao cho 3BD = 2CE.
a) Vẽ (O’) ngoại tiếp tam giác ABC. Gọi M là điểm chính giữa cung BC (M và A nằm khác phía
đối với BC). I là điểm trên cạnh BC và 3 2BI IC , MI cắt (O’) tại N khác M. Chứng minh rằng
tâm O của đường tròn ngoại tiếp tam giác ADE thuộc một đường thẳng cố định.
Bài 53.Từ điểm A nằm ngoài đường tròn (O) vẽ các tiếp tuyến AM, AN đến đường tròn (O) và
cát tuyến ADC. B là điểm trên cung CM (không chứa D). Gọi H là giao điểm của MN và BD, E
là giao điểm của CH và đường tròn (O). Chứng minh rằng A, E, B thẳng hàng.
Luyện thi học sinh giỏi, thi chuyên toán lớp 10 Thầy Hồng Trí Quang
20
Bài 54.Cho đường tròn (O) có dây cung BC (khác đường kính) cố định, A là điểm chuyển động
trên cung lớn BC, M là trung điểm dây BC. Gọi D là giao điểm của AM và cung nhỏ BC, N là
giao điểm của AB và CD. Chứng minh rằng N thuộc một đường thẳng cố định.
Bài 55.*Cho tam giác ABC vuông tại A có AM là đường trung tuyến, AH là đường cao. Trên tia
đối của tia AM lấy điểm P (P khác A). Các đường thẳng qua H vuông góc với AB và AC lần
lượt cắt các đường thẳng PB và PC tại Q và R tương ứng. Chứng minh A là trực tâm tam giác
PQR
Bài 56.*Cho tam giác ABC nhọn, ba đường cao AD, BE và CF cắt nhau tại H. Qua A vẽ các
đường thẳng song song với BE, CF lần lượt cắt các đường thẳng CF, BE tại P và Q. Chứng minh
rằng PQ vuông góc với trung tuyến AM của tam giác ABC.
ỨNG DỤNG TỨ GIÁC NỘI TIẾP
Ứng dụng hệ thức để chứng minh đẳng thức
Hệ thức 1. Cho đường tròn (O;R) và điểm A nằm ngoài đường tròn. Kẻ tiếp tuyến AB (B là tiếp
điểm) và các cát tuyến ACD, AEF với đường tròn. Chứng minh rằng
2 2 2
. .AFAB AC AD AE AO R    (*)
Hệ thức 2. Cho đường tròn (O;R) và một điểm A nằm trong đường tròn  A O . Qua A kẻ hai
dây cung CD và EF. Chứng minh rằng 2 2
. .AFAC AD AE R OA   (**)
Các hệ thức (*) và (**) được gọi là hệ thức lượng giác trong đường tròn. Bạn đọc có thể chứng
minh chúng bằng kiến thức của tam giác đông dạng. vận dụng hệ thức lượng đó ta sẽ giải quyết
được nhiều bài toán về chứng minh đẳng thức và bất đẳng thức trong đường tròn. Sau đây là một
thí dụ minh họa.
Bài 57. Cho tam giác ABC với I, J lần lượt là tâm đường tròn nội tiếp, tâm đường tròn bàng tiếp
góc A. chứng minh rằng .AJ .AI AB AC .
Lời giải.
Ta có ICJ 90o
IBJ  
A
J
B C
I
O
D
Luyện thi học sinh giỏi, thi chuyên toán lớp 10 Thầy Hồng Trí Quang
21
Suy ra tứ giác IBJC nội tiếp đường tròn đường kính IJ, tâm O (trung điểm IJ). Trên tia AB lấy
điểm D sao cho AD AC . Tam giác ACD cân tại A nên AJ là trung trực của CD, suy ra
OD OC . Vậy D thuộc đường tròn (O).
Áp dụng hệ thức (*) ta có .AJ .AI AD AB , mà AD AC nên .AJ .AI AB AC (đpcm)
Bài 58. Cho tam giác ABC với hai đường phân giác trong và ngoài của góc BAC lần lượt là AD
và AE. Chứng minh rằng 2 2
. . D .AB AC DB DC A EB EC AE   
Lời giải.
Giả sử O là tâm của đường tròn ngoại tiếp tam giác ABC. Tia AD cắt đường tròn ở M (điểm
chính giữa cung BC). Tia đối của tia AE cắt đường tròn tại N. dễ thấy 90o
MAN  nên MN là
đường kính của đường tròn (O). Suy ra MN BC . Xét hai tam giác AMB và ADC có
1 2 ( ),A A gt AMB ACD  (góc nội tiếp cùng chắn cung AB), suy ra AMB ACD  (g.g)
Nên . .
AB AM
AB AC AM AD
AD AC
   hay   2
. . .AB AC AD DM AD AD DM AD   
Áp dụng hệ thức (***) với hai dây cung AM và BC ta có . .DM AD DB DC .
Do đó 2
. . DAB AC DB DC A  (1)
Hai tam giác ANB và ACE có 3 4A A (cùng phụ với hai góc bằng nhau 1 2A A );
 ABN AEC AMN  , suy ra ANB ACE  (g.g) nên
AB AN
AE AC

 
2
. . .
. .
AB AC AN AE EN EA AE
AB AC EN AE AE
   
  
Áp dụng hệ thức (*) với hai cát tuyến EAN và ECB ta có . .EN AE EB EC , do đó
2
. .AB AC EB EC AE  (2)
Kết hợp (1) và (2) ta có đpcm
4
2
1
3
D
E
N
M
B
A
C
Luyện thi học sinh giỏi, thi chuyên toán lớp 10 Thầy Hồng Trí Quang
22
Bài 59. Hệ thức Euler. Cho tam giác ABC nội tiếp đường tròn (O;R). Gọi I là tâm và r là bán
kính đường tròn nội tiếp, J là tâm và or là bán kính đường tròn bàng tiếp góc A của tam giác
ABC. Chứng minh rằng 2 2
2OI R Rr  và 2 2
2 oOJ R Rr  .
Lời giải.
Theo thí dụ 1, ta có tứ giác IBJC nội tiếp đường tròn đường kính Ị, tâm M là trung điểm IJ đồng
thời là điểm chính giữa của cung nhỏ BC của đường tròn (O) nên MB MI MJ 
Gọi D và E lần lượt là hình chiếu vuông góc của I và J xuống đường thẳng AB, còn MN là
đường kính của đường tròn (O).
Xét hai tam giác vuông IAD và MNB có 1A N (góc nộ tiếp cùng chắn cung MB) nên đồng
dạng, suy ra
IA ID
MN MB
 . . . 2IAMB IAMI ID MN Rr   
Áp dụng hệ thức (**) đối với dây cung AIM ta có 2 2
.IA IM R OI  , suy ra 2 2
2R OI Rr  hay
2 2
2OI R Rr 
Tương tự, hai tam giác vuông JAE và MNB đồng dạng nên ta có
JA JE
MN MB
 hay
. . 2 . a
JA JE
JA MJ JE MN R r
MN MJ
    .
Áp dụng hệ thức (*) đối với cát tuyến JMA, ta có 2 2
.JA JM OJ R  . Từ đó suy ra
2 2 2 2
2 . 2 .a aOJ R R r OJ R R r    
Tự luyện
Bài 60. Cho tam giác ABC (các góc B, C đều nhọn), các đường cao BD, CE cắt nhau tại H.
chứng minh rằng 2
. .BH BD CH CE BC 
D
E
N
M
O
B
A
C
J
Luyện thi học sinh giỏi, thi chuyên toán lớp 10 Thầy Hồng Trí Quang
23
Bài 61. Cho M là một điểm tùy ý thuộc đường thẳng cố định d nằm ngoài đường tròn (O;R). Từ
M kẻ hai tiếp tuyến MP và MQ với đường tròn (O) trong đó P, Q là các tiếp điểm. Hạ OH vuông
góc với đường thẳng d. Dây cung PQ cắt OH ở I, cắt OM ở K. Chứng minh rằng
a) 2
. .OI OH OK OM R  .
b)Khi M thay đổi trên đường thẳng d thì vị trí của điểm I luôn cố định.
Bài 62. Từ một điểm M nằm ngoài đường tròn (O;R) vẽ hai tiếp tuyến MA, MB (A, B là tiếp
điểm) và một cát tuyến qua M cắt đường tròn tại C, D (C nằm giữa M và D). Gọi E là giao điểm
của AB và OM. Khi cung CAD nhỏ hơn cung CBD. Chứng minh rằng 2DEC DBC .
Lời giải.
Áp dụng hệ thức (*) ta có 2
.MB MC MD . Trong tam giác vuông OBM có BE là đường cao nên
2
. . .MB ME MO MC MD ME MO   .
Suy ra ( . . )MEC MDO c g c MEC MDO    tứ giác CDOE nội tiếp DOC DEC  .
Mà 2DOC DBC (cùng chắn cung DC của đường tròn (O)) nên 2DEC DBC .
Bài 63. Cho đường tròn (O) đường kính AB. Vẽ đường tròn tâm A cắt đường tròn (O) ở C và D.
Kẻ dây BN của đường tròn(O), cắt đường tròn (A) tại điểm E ở bên trong đường tròn (O).
Chứng minh rằng 2
.NE NC ND .
Bài 64. Cho tứ giác ABCD. Các đường thẳng AD, BC và AB, CD lần lượt cắt nhau tại E và F.
Chứng minh rằng điều kiện cần và đủ để tứ giác ABCD nội tiếp đường tròn là
2
. .EA ED FA FB EF  .
Lời giải.
E
C
M
O
A
B
D
Luyện thi học sinh giỏi, thi chuyên toán lớp 10 Thầy Hồng Trí Quang
24
Giả sử đường tròn ngoại tiếp tam giác ABE cắt EF tại M, đường tròn ngoại tiếp tam giác ADF
cắt EF tại N. Áp dụng hệ thức (*) với hai cát tuyến EAD, ENF và hai cát tuyến FAB; FMN ta có
. .EF
. .
EA ED EN
FA FB FM EF


(1)
(2)
Từ (1) và (2) có  . .EA ED FA FB EF EN FM   (3)
Giả sử tứ giác ABCD nội tiếp, ta có ABC ADF (cùng bù với góc ADC ), mà ABC AME
(cùng bù với góc ABE ) nên AME ADF .
Suy ra tứ giác AMFD nội tiếp, chứng tỏ M N .
Từ (3) suy ra 2
. . EFEA ED FA FB  (4)
Ngược lại, giả sử có (4), kết hợp với (3), suy ra EF EN FM  . Chứng tỏ M N .
Từ đó ;AME ABC AME ADF  , suy ra ADF ABC nên 180o
ADC ABC  hay tứ giác
ABCD nội tiếp đường tròn.
Định lí P tô lê mê và ứng dụng
Định lí Ptô-lê-mê có thể phát biểu thành định lý thuận và đảo:
-Thuận:Nếu một tứ giác nội tiếp trong một đường tròn thì tích của hai đường chéo bằng tổng
các tích của các cặp cạnh đối diện.
- Đảo:Nếu một tứ giác thỏa mãn điều kiện tổng các tích của các cặp cạnh đối diện bằng tích của
hai đường chéo thì tứ giác đó nội tiếp một đường tròn. *]
Ứng dụng định lí Ptoleme để chứng minh đẳng thức
Bài 65. Cho tam giác đều ABC nội tiếp đường tròn tâm O bán kính R. Gọi M là điểm bất kì
thuộc cung BC
C
E
F
D
B
M
A
N
Luyện thi học sinh giỏi, thi chuyên toán lớp 10 Thầy Hồng Trí Quang
25
a) Chứng minh rằng MA = MB + MC
b) Gọi D là giao điểm của MA và BC. Chứng minh rằng:
1 1 1
MB MC MD
 
c) * Tính tổng 2 2 2
MA MB MC  theo R.
HD b) MCD đồng dạng MAB; c) Đặt MA = x, MB = y, Tổng = 2 2 2
2( )x y xy 
Kẻ BH vuông góc AM; 2MH = y nên 2 2 2 2 2 2 2
2 2( ) 2( ) 6RAB AH BH x y xy     
(Đề thi vào trường THPT chuyên Lê Quí Đôn, Quảng Trị, năm học 2005-2006)
Bài 66. Cho tam giác đều ABC có các cạnh bằng a.Trên AC lấy điểm Q di động, trên tia đối
của tia CB lấy điểm P di động sao cho 2
.AQ BP a . Gọi M là giao điểm của BQ và AP. Chứng
minh rằng: AM MC BM 
Chứng minh:
Từ giả thiết 2
. . .ABAQ BP a AQ BP AB   .
Xét ΔABQ  ΔBPA(c.g.c) ABQ=APB (1)
Lại có ABQ+MBP=600
(2)
Từ: (1), (2) ⇒BMP=1800
− MBP−MPB =1200
AMB=1800
−BMP=1800
−1200
= 600
=ACB.
 tứ giác AMCB nội tiếp được đường tròn.
Áp dụng định lí Ptô-lê-mê cho tứ giác AMCB nội tiếp
và giả thiết AB = BC = CA ta có:
. . .AB MC BC AM BM AC AM MC BM     (đpcm)
Bài 67. Tam giác ABC vuông có BC > CA > AB. Gọi D là một điểm trên cạnh BC, E là một
điểm trên cạnh AB kéo dài về phía điểm A sao cho BD = BE = CA. Gọi P là một điểm trên
cạnh AC sao cho E, B, D, P nằm trên một đường tròn. Q là giao điểm thứ hai của BP với đường
tròn ngoại tiếp  ABC. Chứng minh rằng: AQ + CQ = BP
(Đề thi chọn đội tuyển Hồng Kông tham dự IMO 2000, HongKong TST 2000)
Dựa vào các đại lượng trong tam giác bằng nhau theo giả thiết, ta sử dụng ĐL tam giác đồng
dạng để suy ra các tỉ số liên quan và sử dụng phép thế để suy ra điều phải chứng minh.
Chứng minh:
Xét các tứ giác nội tiếp ABCQ và BEPD ta có:
CAQ=CBQ=DEP (cùng chắn các cung tròn)
Luyện thi học sinh giỏi, thi chuyên toán lớp 10 Thầy Hồng Trí Quang
26
Mặt khác AQC=1080
−ABC=EPD
Xét ΔAQC và ΔEPDcó: AQC=EPD; CAQ=DEP
 ΔAQCΔEPD  AQ/EP=CA/ED
 AQ.ED =EP.CA=EP.BD( 1) (do AC=BD)
AC/ED=QC/PD  ED.QC=AC.PD=BE.PD (2) (do AC=BE)
Áp dụng định lí Ptô-lê-mê cho tứ giác nội tiếp BEPD ta có:
EP.BD+BE.PD=ED.BP (3)
Từ (1), (2), (3) suy ra:
AQ.ED + QC.ED = ED.BP  AQ+QC=BP (đpcm)
Bài 68. Định lí Carnot
Cho tam giác nhọn ABC nội tiếp trong đường tròn (O,R) và ngoại tiếp đường tròn (I,r). Gọi x, y,
z lần lượt là khoảng cách từ O tới các cạnh tam giác. Chứng minh rằng: x y z R r   
Chứng minh:
Gọi M,N,P lần lượt là trung điểm của BC,CA,AB.
Giả sử x = OM, y = ON, z = OP, BC = a, CA = b, AB = c.
Tứ giác OMBP nội tiếp, theo đẳng thức Ptô-lê-mê ta có:
. . .OB PM OP MB OM PB 
Do đó: 2 2 2
. . 1(. )Rb z a x c 
Tương tự ta cũng có 2 2 2
. . .R c y a xb 
(2) và
2 2 2
. . .R a y c z b  (3)
Mặt khác:
2 2 2 2 2 2
( ) . .r a b c SABC SOBC SOCA SOAB x a yb z c         (4)
Từ (1), (2), (3), (4) ta có: 2 2
( )( ) ( )( )R r a b c x y z a b c            R r x y z
(Bài này còn gọi là Định lí Carnot- 1 định lí khá là quen thuộc và cách chứng minh cũng khá
đơn giản. Ứng dụng của định lí này như đã nói là dùng nhiều trong tính toán các đại lượng
trong tam giác)
Ứn dụng Carnot:
Bài 69. *Ứng dụng Định lí Carnot
Cho tam giác ABC cân tại A, nội tiếp trong đường tròn (O) bán kính R = 1, có AB = 2.BC. Tính
bán kính đường tròn nội tiếp (O’) nội tiếp tam giác ABC?
Luyện thi học sinh giỏi, thi chuyên toán lớp 10 Thầy Hồng Trí Quang
27
Sử dụng tam giác đồng dạng, r = 3/8
R + r =OI + OK + OH (khoảng cách đến 3 canh). Sử dụng diện tích;
Chứng minh AO’ = 4r bằng tam giác đồng dạng;
OI, OK, OH vuông
Bài 70. Cho tam giác ABC nội tiếp trong đường tròn (O) và AC=2AB. Các đường thẳng tiếp
xúc với đường tròn (O) tại A,C cắt nhau ở P. Chứng minh rằng BP đi qua điểm chính giữa của
cung BAC.
Chứng minh:
Gọi giao điểm của BP với đường tròn là N. Nối AN,NC.
Xét NPCvà CPB có: PCNˆ=PBCˆ,Pˆ chung
 ΔNPC  ΔCPB(g.g)  PC/PB=NC/BC(1)
Tương tự ta cũng có ΔPAN  ΔPBA(g.g)
 AP/BP=AN/AB (2)
Mặt khác PA=PC (2)
 PA/PB=NC/BC=AN/AB
 NC.AB=BC.AN(3)
Áp dụng định lí Ptô-lê-mê cho tứ giác nội tiếp ABCN ta có:
AN.BC+AB.NC=AC.BN
Từ (3) 2AB.NC=AC.BN=2AB.BN
NC=BN. (đpcm)
Bài 71. Cho tam giác ABC có I là tâm đường tròn nội tiếp, O là tâm đường tròn ngoại tiếp và
trọng tâm G. Giả sử rằng 𝑂𝐼𝐴̂ = 900
. Chứng minh rằng IG song song với BC.
Chứng minh
Kéo dài AI cắt (O) tại N. Khi đó N là điểm chính
giữa cung BC (không chứa A).
Ta có: BN=NC(1). Lại có:
Luyện thi học sinh giỏi, thi chuyên toán lớp 10 Thầy Hồng Trí Quang
28
IBNˆ= BINˆ BN=IN(2)
Do OI  AI suy ra IA=IN=1/2 sđ cung BC(3)
Từ (1),(2),(3) BN=NC=IN=IA(4)
Áp dụng định lí Ptô-lê-mê cho tứ giác nội tiếp ABNC ta có:
BN.AC+AB.NC=BC.AN
Từ (4)
BN(AC+AB)=2BN.BC AC+AB=2BC (5)
Áp dụng tính chất đường phân giác trong tam giác và (5) ta có:
AB/BD=IA/ID=AC/CD=(AB+AC)/(BD+CD)=(AB+AC)/(BC)=2BC/BC=2
Vậy IA/ID=2(6)
Mặt khác G là trọng tâm của tam giác suy ra AG/GM=2(7)
Từ (6),(7) IA/ID=2=AG/GM
Suy ra IG là đường trung bình của tam giác ADM hay IG song song với BC. (đpcm)
Bài 72. Cho tam giác ABC nội tiếp đường tròn (O), CM là trung tuyến. Các tiếp tuyến tại A
và B của (O) cắt nhau ở D. Chứng minh rằng: ACD BCM
Chứng minh:
Gọi N là giao điểm của CD với (O). Xét tam giác DNB
và DBC có:
DBNˆ=DCBˆ,Dˆ chung.
ΔDBN  ΔDCB(g.g) NB/CB=BD/CD(1)
Tương tự ta cũng có :
ΔDNA  ΔDAC(g.g) NA/AC=DA/CD(2)
Mà BD=DAnên từ (1),(2)NB/CB=NA/AC  NB.AC=AN.BC(3)
Áp dụng định lí Ptô-lê-mê cho tứ giác nội tiếp ANBC ta có:
AN.BC+BN.AC=AB.NC
Từ (3) và giả thiết
AB=2BM 2AN.BC=2BM.NC AN/NC=BM/BC
Luyện thi học sinh giỏi, thi chuyên toán lớp 10 Thầy Hồng Trí Quang
29
Xét ΔBMC và ΔNAC có:
MBCˆ=ANCˆ, AN/NC=BM/BC ΔBMC ΔNAC(c.g.c) BCMˆ=NACˆ
Vậy bài toán được chứng minh.
Ứng dụng định lí Ptoleme để chứng minh đẳng thức
Bài 73. Cho tứ giác nội tiếp có các cạnh liên tiếp bằng a, b, c, d và các đường chéo bằng p, q.
Chứng minh rằng:   2 2 2 2
pq a b c d  
HD Bài toán đơn giản, sử dụng Ptoleme và Bunhia
Cho đường tròn (O)) và BC là một dây cung khác đường kính của đường tròn. Tìm điểm A
thuộc cung lớn BC sao cho AB + AC lớn nhất.
Gọi D là điểm chính giữa của cung nhỏ BC thì DB = DC = a không đổi. Áp dụng Ptoleme thì A
là là điểm chính giữa cung lớn BC.
Tự luyện
Bài 74. (149T5/246 – 45 năm).Cho tam giác ABC, biết rằng: 𝐴̂ = 2𝐵̂ = 4𝐶̂. Chứng minh
rằng:
1
𝐴𝐵
=
1
𝐵𝐶
+
1
𝐴𝐶
HD: Trên cung BC lấy D sao cho hai cung AB = BD. Áp dụng định lí Ptoleme
Bài 75. Cho tứ giác ABC nội tiếp đường tròn (O). Chứng minh:
. .
. .
AC BC CD AB BD
BD BC BA DC DA



Lấy E, F thuộc đường tròn sao cho: ;CDB ADE BDA DCF 
Áp dụng định lí cho tứ giác AECD và BCDF. Chứng minh ED = FC (do 2 cung bằng nhau) ta
có đpcm
Bài 76. Cho tam giác ABC với BE, CF là các đường phân giác trong. Các tia EF, FE cắt
đường tròn ngoại tiếp tam giác theo thứ tự tại M và N. Chứng minh rằng:
1 1 1 1 1 1
BM CN AM AN BN CM
    
Luyện thi học sinh giỏi, thi chuyên toán lớp 10 Thầy Hồng Trí Quang
30
HD Đặt BC = a, CA = b, AB = c.
áp dụng đli cho tứ giác AMBC, ANCB. Mặt khác ;
AM MF AN AF
BN BF BM MF
 
Theo tính chất phân giác ta có:
.
.
AM AN AF b
BM BN BF a
 
Tương tự:
.
.
AM AN AE c
BM BN CE a
 
Bài 77. Giả sử M, N là các điểm nằm trong tam giác ABC sao cho ;MAB NAC MBA NBC  .
Chứng minh rằng:
. . .
1
. . .CB
AM AN BM BN CM CN
AB AC BA BC CA
  
HD lấy K trên đường thẳng BN sao cho BCK BMA ;
AB BK
MB BC
 
Mặt khác:
AB BK AK
MB BC CM
 
Từ đó tính được CK, AK, BK. Thay vào định lí Ptoleme trong tứ giác ABCK ta có:
. . .AC NK AN CK CN AK 
.
.( ) . . .
AB BC
AC BK BN AN CK CN AK AC BN
BM
 
    
 
Luyện thi học sinh giỏi, thi chuyên toán lớp 10 Thầy Hồng Trí Quang
31
Bài 78. (CMO 1988, Trung Quốc)
ABCD là một tứ giác nội tiếp với đường tròn ngoại tiếp có tâm ) và bán kính R. Các tia AB, BC,
CD, DA cắt (O,2R) lần lượt tại A′, B′, C′, D′. Chứng minh rằng:
( )2A B B C C D D A AB BC CD DA           
Bài 79. Cho đường tròn (O) và dây cung BC khác đường kính. Tìm điểm A thuộc cung
lớn BC của đường tròn để AB+2AC đạt giá trị lớn nhất.
Bài 80. Cho tam giác ABC nội tiếp đường tròn (O). Đường tròn (O′) nằm trong (O) tiếp xúc
với (O) tại T thuộc cung AC (ko chứa B). Kẻ các tiếp tuyến AA′,BB′,CC′tới (O′). Chứng minh
rằng: BB′.AC=AA′.BC+CC′.AB
Ứng dụng tứ giác nội tiếp Chứng minh ba điểm thẳng hàng
Updating…
Chứng minh dựa vào góc
Chứng minh dựa vào song song
Bài 81. Cho tam giác ABC nội tiếp đường tròn (O) và I là tâm đường tròn nội tiếp. Gọi M,
N, D là điểm chính giữa các cung nhỏ BC, CA, AB. Giả sử MN cắt AC tại Q; MD cắt AB tại
P. Chứng minh rằng:
a) MB = MI
b) Ba điểm P, I, Q thẳng hàng.
HD a) nếu I thuộc AD thì I là tâm nội tiếp khi và chỉ khi MB = MI. b) Từ câu a suy ra MP là
trung trực BI, suy ra góc PBI = góc PIB, suy ra IP song song BC. Tương tự IQ song song BC
Ứng dụng tứ giác nội tiếp Chứng minh ba đường đồng quy
Updating…
Luyện thi học sinh giỏi, thi chuyên toán lớp 10 Thầy Hồng Trí Quang
32

More Related Content

What's hot

chuyen de hinh hoc khong gian 11 co loi giai - bien soan cong phu - hay
chuyen de hinh hoc khong gian 11 co loi giai - bien soan cong phu - haychuyen de hinh hoc khong gian 11 co loi giai - bien soan cong phu - hay
chuyen de hinh hoc khong gian 11 co loi giai - bien soan cong phu - hay
Hoàng Thái Việt
 
19 phương phap chứng minh bất đẳng thức
19 phương phap chứng minh bất đẳng thức19 phương phap chứng minh bất đẳng thức
19 phương phap chứng minh bất đẳng thứcThế Giới Tinh Hoa
 
đề cương ôn tập và kiểm tra chương 2 phân thức toán 8
đề cương ôn tập và kiểm tra chương 2 phân thức toán 8đề cương ôn tập và kiểm tra chương 2 phân thức toán 8
đề cương ôn tập và kiểm tra chương 2 phân thức toán 8
Hoàng Thái Việt
 
Chuyên đề, phương pháp tìm số hạng tổng quát của dãy số
Chuyên đề, phương pháp tìm số hạng tổng quát của dãy sốChuyên đề, phương pháp tìm số hạng tổng quát của dãy số
Chuyên đề, phương pháp tìm số hạng tổng quát của dãy sốThế Giới Tinh Hoa
 
Các bài toán về tỷ lệ thức
Các bài toán về tỷ lệ thứcCác bài toán về tỷ lệ thức
Các bài toán về tỷ lệ thức
Kim Liên Cao
 
Tuyển tập các bài Toán Hình học lớp 9 ôn thi vào 10
Tuyển tập các bài Toán Hình học lớp 9 ôn thi vào 10Tuyển tập các bài Toán Hình học lớp 9 ôn thi vào 10
Tuyển tập các bài Toán Hình học lớp 9 ôn thi vào 10
BOIDUONGTOAN.COM
 
50 bài toán casio số phức nâng cao
50 bài toán casio số phức nâng cao50 bài toán casio số phức nâng cao
50 bài toán casio số phức nâng cao
Lâm Trần Khắc
 
Bồi dưỡng nâng cao HSG Toán lớp 7 qua 16 chuyên đề - Thầy Thích
Bồi dưỡng nâng cao HSG Toán lớp 7 qua 16 chuyên đề - Thầy ThíchBồi dưỡng nâng cao HSG Toán lớp 7 qua 16 chuyên đề - Thầy Thích
Bồi dưỡng nâng cao HSG Toán lớp 7 qua 16 chuyên đề - Thầy Thích
Bồi dưỡng Toán lớp 6
 
Chuyên dề dấu tam thức bậc hai
Chuyên dề dấu tam thức bậc haiChuyên dề dấu tam thức bậc hai
Chuyên dề dấu tam thức bậc hai
Nhập Vân Long
 
Cđ một số ứng dụng định lí mê nê la uýt và xê va
Cđ một số ứng dụng định lí mê nê la uýt và xê vaCđ một số ứng dụng định lí mê nê la uýt và xê va
Cđ một số ứng dụng định lí mê nê la uýt và xê va
Cảnh
 
Sử Dụng Hàng Điểm Điều Hòa Trong Giải Toán Hình Học Phẳng.doc
Sử Dụng Hàng Điểm Điều Hòa Trong Giải Toán Hình Học Phẳng.docSử Dụng Hàng Điểm Điều Hòa Trong Giải Toán Hình Học Phẳng.doc
Sử Dụng Hàng Điểm Điều Hòa Trong Giải Toán Hình Học Phẳng.doc
DV Viết Luận văn luanvanmaster.com ZALO 0973287149
 
Chuyên đề giá trị tuyệt đối
Chuyên đề giá trị tuyệt đốiChuyên đề giá trị tuyệt đối
Chuyên đề giá trị tuyệt đối
youngunoistalented1995
 
Một số bài hình 9 chọn lọc
Một số bài hình 9 chọn lọcMột số bài hình 9 chọn lọc
Một số bài hình 9 chọn lọc
Ngo Quang Viet
 
CHUYÊN ĐỀ HÌNH HỌC ÔN THI VÀO LỚP 10 CÁC TRƯỜNG CHUYÊN
CHUYÊN ĐỀ HÌNH HỌC ÔN THI VÀO LỚP 10 CÁC TRƯỜNG CHUYÊNCHUYÊN ĐỀ HÌNH HỌC ÔN THI VÀO LỚP 10 CÁC TRƯỜNG CHUYÊN
CHUYÊN ĐỀ HÌNH HỌC ÔN THI VÀO LỚP 10 CÁC TRƯỜNG CHUYÊN
BOIDUONGTOAN.COM
 
110 bài hình học về phương trình đường thẳng
110 bài hình học về phương trình đường thẳng 110 bài hình học về phương trình đường thẳng
110 bài hình học về phương trình đường thẳng
Hades0510
 
Chứng minh đẳng thức vectơ và phân tích vectơ
Chứng minh đẳng thức vectơ và phân tích vectơ Chứng minh đẳng thức vectơ và phân tích vectơ
Chứng minh đẳng thức vectơ và phân tích vectơ
Jackson Linh
 
Bai tap-hinh-lop-7-hay-co-loi-giai
Bai tap-hinh-lop-7-hay-co-loi-giaiBai tap-hinh-lop-7-hay-co-loi-giai
Bai tap-hinh-lop-7-hay-co-loi-giai
Sa Hong
 
Toán lớp 9 - Tổng hợp kiến thức lý thuyết Đại số 9 và Hình học 9
Toán lớp 9 - Tổng hợp kiến thức lý thuyết Đại số 9 và Hình học 9Toán lớp 9 - Tổng hợp kiến thức lý thuyết Đại số 9 và Hình học 9
Toán lớp 9 - Tổng hợp kiến thức lý thuyết Đại số 9 và Hình học 9
Bồi dưỡng Toán lớp 6
 

What's hot (20)

chuyen de hinh hoc khong gian 11 co loi giai - bien soan cong phu - hay
chuyen de hinh hoc khong gian 11 co loi giai - bien soan cong phu - haychuyen de hinh hoc khong gian 11 co loi giai - bien soan cong phu - hay
chuyen de hinh hoc khong gian 11 co loi giai - bien soan cong phu - hay
 
19 phương phap chứng minh bất đẳng thức
19 phương phap chứng minh bất đẳng thức19 phương phap chứng minh bất đẳng thức
19 phương phap chứng minh bất đẳng thức
 
đề cương ôn tập và kiểm tra chương 2 phân thức toán 8
đề cương ôn tập và kiểm tra chương 2 phân thức toán 8đề cương ôn tập và kiểm tra chương 2 phân thức toán 8
đề cương ôn tập và kiểm tra chương 2 phân thức toán 8
 
Chuyên đề, phương pháp tìm số hạng tổng quát của dãy số
Chuyên đề, phương pháp tìm số hạng tổng quát của dãy sốChuyên đề, phương pháp tìm số hạng tổng quát của dãy số
Chuyên đề, phương pháp tìm số hạng tổng quát của dãy số
 
Các bài toán về tỷ lệ thức
Các bài toán về tỷ lệ thứcCác bài toán về tỷ lệ thức
Các bài toán về tỷ lệ thức
 
Bất đẳng thức hình học
Bất đẳng thức hình họcBất đẳng thức hình học
Bất đẳng thức hình học
 
Tuyển tập các bài Toán Hình học lớp 9 ôn thi vào 10
Tuyển tập các bài Toán Hình học lớp 9 ôn thi vào 10Tuyển tập các bài Toán Hình học lớp 9 ôn thi vào 10
Tuyển tập các bài Toán Hình học lớp 9 ôn thi vào 10
 
50 bài toán casio số phức nâng cao
50 bài toán casio số phức nâng cao50 bài toán casio số phức nâng cao
50 bài toán casio số phức nâng cao
 
Bồi dưỡng nâng cao HSG Toán lớp 7 qua 16 chuyên đề - Thầy Thích
Bồi dưỡng nâng cao HSG Toán lớp 7 qua 16 chuyên đề - Thầy ThíchBồi dưỡng nâng cao HSG Toán lớp 7 qua 16 chuyên đề - Thầy Thích
Bồi dưỡng nâng cao HSG Toán lớp 7 qua 16 chuyên đề - Thầy Thích
 
Chuyên dề dấu tam thức bậc hai
Chuyên dề dấu tam thức bậc haiChuyên dề dấu tam thức bậc hai
Chuyên dề dấu tam thức bậc hai
 
Cđ một số ứng dụng định lí mê nê la uýt và xê va
Cđ một số ứng dụng định lí mê nê la uýt và xê vaCđ một số ứng dụng định lí mê nê la uýt và xê va
Cđ một số ứng dụng định lí mê nê la uýt và xê va
 
Sử Dụng Hàng Điểm Điều Hòa Trong Giải Toán Hình Học Phẳng.doc
Sử Dụng Hàng Điểm Điều Hòa Trong Giải Toán Hình Học Phẳng.docSử Dụng Hàng Điểm Điều Hòa Trong Giải Toán Hình Học Phẳng.doc
Sử Dụng Hàng Điểm Điều Hòa Trong Giải Toán Hình Học Phẳng.doc
 
Chuyên đề giá trị tuyệt đối
Chuyên đề giá trị tuyệt đốiChuyên đề giá trị tuyệt đối
Chuyên đề giá trị tuyệt đối
 
Một số bài hình 9 chọn lọc
Một số bài hình 9 chọn lọcMột số bài hình 9 chọn lọc
Một số bài hình 9 chọn lọc
 
Bdt thuần nhất
Bdt thuần nhấtBdt thuần nhất
Bdt thuần nhất
 
CHUYÊN ĐỀ HÌNH HỌC ÔN THI VÀO LỚP 10 CÁC TRƯỜNG CHUYÊN
CHUYÊN ĐỀ HÌNH HỌC ÔN THI VÀO LỚP 10 CÁC TRƯỜNG CHUYÊNCHUYÊN ĐỀ HÌNH HỌC ÔN THI VÀO LỚP 10 CÁC TRƯỜNG CHUYÊN
CHUYÊN ĐỀ HÌNH HỌC ÔN THI VÀO LỚP 10 CÁC TRƯỜNG CHUYÊN
 
110 bài hình học về phương trình đường thẳng
110 bài hình học về phương trình đường thẳng 110 bài hình học về phương trình đường thẳng
110 bài hình học về phương trình đường thẳng
 
Chứng minh đẳng thức vectơ và phân tích vectơ
Chứng minh đẳng thức vectơ và phân tích vectơ Chứng minh đẳng thức vectơ và phân tích vectơ
Chứng minh đẳng thức vectơ và phân tích vectơ
 
Bai tap-hinh-lop-7-hay-co-loi-giai
Bai tap-hinh-lop-7-hay-co-loi-giaiBai tap-hinh-lop-7-hay-co-loi-giai
Bai tap-hinh-lop-7-hay-co-loi-giai
 
Toán lớp 9 - Tổng hợp kiến thức lý thuyết Đại số 9 và Hình học 9
Toán lớp 9 - Tổng hợp kiến thức lý thuyết Đại số 9 và Hình học 9Toán lớp 9 - Tổng hợp kiến thức lý thuyết Đại số 9 và Hình học 9
Toán lớp 9 - Tổng hợp kiến thức lý thuyết Đại số 9 và Hình học 9
 

Similar to 9 tu giac noi tiep

9 tu giac noi tiep htq
9 tu giac noi tiep htq9 tu giac noi tiep htq
9 tu giac noi tiep htq
Hồng Quang
 
9 tu giac noi tiep htq
9 tu giac noi tiep htq9 tu giac noi tiep htq
9 tu giac noi tiep htq
Hồng Quang
 
[Duong] chung minh dang thuc tich
[Duong] chung minh dang thuc tich[Duong] chung minh dang thuc tich
[Duong] chung minh dang thuc tich
Zooey Inn
 
các bài toán hình học lớp 9 có lời giải
các bài toán hình học lớp 9 có lời giảicác bài toán hình học lớp 9 có lời giải
các bài toán hình học lớp 9 có lời giải
Khoảnh Khắc Bình Yên
 
9 hinh nang cao htq
9 hinh nang cao htq9 hinh nang cao htq
9 hinh nang cao htq
Hồng Quang
 
Duong tron bttl phan 5 ct1, ct2
Duong tron bttl phan 5 ct1, ct2Duong tron bttl phan 5 ct1, ct2
Duong tron bttl phan 5 ct1, ct2
Hồng Quang
 
ôn hình lớp 9
ôn hình lớp 9ôn hình lớp 9
ôn hình lớp 9
tamhvtc
 
Dau hieu tich chung minh tu giac noi tiep htq
Dau hieu tich chung minh tu giac noi tiep   htqDau hieu tich chung minh tu giac noi tiep   htq
Dau hieu tich chung minh tu giac noi tiep htq
Hồng Quang
 
Duong tron bttl phan 6 ct1, ct2
Duong tron bttl phan 6 ct1, ct2Duong tron bttl phan 6 ct1, ct2
Duong tron bttl phan 6 ct1, ct2
Hồng Quang
 
Goc co dinh ben trong va ben ngoai duong tron
Goc co dinh ben trong va ben ngoai duong tronGoc co dinh ben trong va ben ngoai duong tron
Goc co dinh ben trong va ben ngoai duong tronTu Em
 
Thay khanh iwaz
Thay khanh iwazThay khanh iwaz
Thay khanh iwaz
ChnhTrung3
 
Chuyen de-hinh-hoc-lop-9-hay-chuyen-de-hinh-hoc-lop-9-hay-tong-hop-on-tap-hin...
Chuyen de-hinh-hoc-lop-9-hay-chuyen-de-hinh-hoc-lop-9-hay-tong-hop-on-tap-hin...Chuyen de-hinh-hoc-lop-9-hay-chuyen-de-hinh-hoc-lop-9-hay-tong-hop-on-tap-hin...
Chuyen de-hinh-hoc-lop-9-hay-chuyen-de-hinh-hoc-lop-9-hay-tong-hop-on-tap-hin...
Toán THCS
 
CHUYÊN ĐỀ HÌNH HỌC 8 CHƯƠNG 3 & ĐỀ KIỂM TRA 2018
CHUYÊN ĐỀ HÌNH HỌC 8 CHƯƠNG 3 & ĐỀ KIỂM TRA 2018CHUYÊN ĐỀ HÌNH HỌC 8 CHƯƠNG 3 & ĐỀ KIỂM TRA 2018
CHUYÊN ĐỀ HÌNH HỌC 8 CHƯƠNG 3 & ĐỀ KIỂM TRA 2018
Hoàng Thái Việt
 
9 hinh nang cao hk 1 dap an
9 hinh nang cao hk 1 dap an9 hinh nang cao hk 1 dap an
9 hinh nang cao hk 1 dap an
Hồng Quang
 
Hệ thức lượng và tỉ số lượng giác
Hệ thức lượng và tỉ số lượng giácHệ thức lượng và tỉ số lượng giác
Hệ thức lượng và tỉ số lượng giác
Hồng Quang
 
Bai toan ve goc
Bai toan ve gocBai toan ve goc
Bai toan ve goc
giangkhuat168
 
Bo de hinh hoc thcs
Bo de hinh hoc thcsBo de hinh hoc thcs
Bo de hinh hoc thcs
khanh271295
 
chuyen de hinh hoc vao 10 (full) ttc
 chuyen de hinh hoc vao 10 (full) ttc chuyen de hinh hoc vao 10 (full) ttc
chuyen de hinh hoc vao 10 (full) ttc
Toán THCS
 
bộ đề+ đáp án đề thi học sinh giỏi hình học 8
bộ đề+ đáp án đề thi học sinh giỏi hình học 8bộ đề+ đáp án đề thi học sinh giỏi hình học 8
bộ đề+ đáp án đề thi học sinh giỏi hình học 8Jackson Linh
 
Tuyển tập đề kiểm tra 15p - 1 tiết - Chương 1 - Đoạn Thẳng - Hình học lớp 6
Tuyển tập đề kiểm tra 15p - 1 tiết - Chương 1 - Đoạn Thẳng - Hình học lớp 6Tuyển tập đề kiểm tra 15p - 1 tiết - Chương 1 - Đoạn Thẳng - Hình học lớp 6
Tuyển tập đề kiểm tra 15p - 1 tiết - Chương 1 - Đoạn Thẳng - Hình học lớp 6
Bồi dưỡng Toán lớp 6
 

Similar to 9 tu giac noi tiep (20)

9 tu giac noi tiep htq
9 tu giac noi tiep htq9 tu giac noi tiep htq
9 tu giac noi tiep htq
 
9 tu giac noi tiep htq
9 tu giac noi tiep htq9 tu giac noi tiep htq
9 tu giac noi tiep htq
 
[Duong] chung minh dang thuc tich
[Duong] chung minh dang thuc tich[Duong] chung minh dang thuc tich
[Duong] chung minh dang thuc tich
 
các bài toán hình học lớp 9 có lời giải
các bài toán hình học lớp 9 có lời giảicác bài toán hình học lớp 9 có lời giải
các bài toán hình học lớp 9 có lời giải
 
9 hinh nang cao htq
9 hinh nang cao htq9 hinh nang cao htq
9 hinh nang cao htq
 
Duong tron bttl phan 5 ct1, ct2
Duong tron bttl phan 5 ct1, ct2Duong tron bttl phan 5 ct1, ct2
Duong tron bttl phan 5 ct1, ct2
 
ôn hình lớp 9
ôn hình lớp 9ôn hình lớp 9
ôn hình lớp 9
 
Dau hieu tich chung minh tu giac noi tiep htq
Dau hieu tich chung minh tu giac noi tiep   htqDau hieu tich chung minh tu giac noi tiep   htq
Dau hieu tich chung minh tu giac noi tiep htq
 
Duong tron bttl phan 6 ct1, ct2
Duong tron bttl phan 6 ct1, ct2Duong tron bttl phan 6 ct1, ct2
Duong tron bttl phan 6 ct1, ct2
 
Goc co dinh ben trong va ben ngoai duong tron
Goc co dinh ben trong va ben ngoai duong tronGoc co dinh ben trong va ben ngoai duong tron
Goc co dinh ben trong va ben ngoai duong tron
 
Thay khanh iwaz
Thay khanh iwazThay khanh iwaz
Thay khanh iwaz
 
Chuyen de-hinh-hoc-lop-9-hay-chuyen-de-hinh-hoc-lop-9-hay-tong-hop-on-tap-hin...
Chuyen de-hinh-hoc-lop-9-hay-chuyen-de-hinh-hoc-lop-9-hay-tong-hop-on-tap-hin...Chuyen de-hinh-hoc-lop-9-hay-chuyen-de-hinh-hoc-lop-9-hay-tong-hop-on-tap-hin...
Chuyen de-hinh-hoc-lop-9-hay-chuyen-de-hinh-hoc-lop-9-hay-tong-hop-on-tap-hin...
 
CHUYÊN ĐỀ HÌNH HỌC 8 CHƯƠNG 3 & ĐỀ KIỂM TRA 2018
CHUYÊN ĐỀ HÌNH HỌC 8 CHƯƠNG 3 & ĐỀ KIỂM TRA 2018CHUYÊN ĐỀ HÌNH HỌC 8 CHƯƠNG 3 & ĐỀ KIỂM TRA 2018
CHUYÊN ĐỀ HÌNH HỌC 8 CHƯƠNG 3 & ĐỀ KIỂM TRA 2018
 
9 hinh nang cao hk 1 dap an
9 hinh nang cao hk 1 dap an9 hinh nang cao hk 1 dap an
9 hinh nang cao hk 1 dap an
 
Hệ thức lượng và tỉ số lượng giác
Hệ thức lượng và tỉ số lượng giácHệ thức lượng và tỉ số lượng giác
Hệ thức lượng và tỉ số lượng giác
 
Bai toan ve goc
Bai toan ve gocBai toan ve goc
Bai toan ve goc
 
Bo de hinh hoc thcs
Bo de hinh hoc thcsBo de hinh hoc thcs
Bo de hinh hoc thcs
 
chuyen de hinh hoc vao 10 (full) ttc
 chuyen de hinh hoc vao 10 (full) ttc chuyen de hinh hoc vao 10 (full) ttc
chuyen de hinh hoc vao 10 (full) ttc
 
bộ đề+ đáp án đề thi học sinh giỏi hình học 8
bộ đề+ đáp án đề thi học sinh giỏi hình học 8bộ đề+ đáp án đề thi học sinh giỏi hình học 8
bộ đề+ đáp án đề thi học sinh giỏi hình học 8
 
Tuyển tập đề kiểm tra 15p - 1 tiết - Chương 1 - Đoạn Thẳng - Hình học lớp 6
Tuyển tập đề kiểm tra 15p - 1 tiết - Chương 1 - Đoạn Thẳng - Hình học lớp 6Tuyển tập đề kiểm tra 15p - 1 tiết - Chương 1 - Đoạn Thẳng - Hình học lớp 6
Tuyển tập đề kiểm tra 15p - 1 tiết - Chương 1 - Đoạn Thẳng - Hình học lớp 6
 

More from Hồng Quang

10 de tang hsg quan huyen thay hong tri quang
10 de tang hsg quan   huyen thay hong tri quang10 de tang hsg quan   huyen thay hong tri quang
10 de tang hsg quan huyen thay hong tri quang
Hồng Quang
 
Pt co ban bttl phan 6 ct0, ct1, ct2
Pt co ban bttl phan 6 ct0, ct1, ct2Pt co ban bttl phan 6 ct0, ct1, ct2
Pt co ban bttl phan 6 ct0, ct1, ct2
Hồng Quang
 
9 pp danh gia
9 pp danh gia9 pp danh gia
9 pp danh gia
Hồng Quang
 
9 can thuc nc
9 can thuc nc9 can thuc nc
9 can thuc nc
Hồng Quang
 
20 cach cm bdt nesbit
20 cach cm bdt nesbit20 cach cm bdt nesbit
20 cach cm bdt nesbit
Hồng Quang
 
9 drichle
9 drichle9 drichle
9 drichle
Hồng Quang
 
[Htq] toan 9 hsg tp hn dap an
[Htq] toan 9 hsg tp hn   dap an[Htq] toan 9 hsg tp hn   dap an
[Htq] toan 9 hsg tp hn dap an
Hồng Quang
 
9 [htq] de thi hsg 3 2 lopluyenthi
9 [htq] de thi hsg 3 2 lopluyenthi9 [htq] de thi hsg 3 2 lopluyenthi
9 [htq] de thi hsg 3 2 lopluyenthi
Hồng Quang
 
9 03 de thi tet 2016
9 03 de thi tet 20169 03 de thi tet 2016
9 03 de thi tet 2016
Hồng Quang
 
9 pt vo ti (co ban) htq
9 pt vo ti (co ban) htq9 pt vo ti (co ban) htq
9 pt vo ti (co ban) htq
Hồng Quang
 
Đồ thị hàm số - toán lớp 9
Đồ thị hàm số - toán lớp 9Đồ thị hàm số - toán lớp 9
Đồ thị hàm số - toán lớp 9
Hồng Quang
 
Ba dạng hệ phương trình cơ bản
Ba dạng hệ phương trình cơ bảnBa dạng hệ phương trình cơ bản
Ba dạng hệ phương trình cơ bản
Hồng Quang
 
9 pt nghiem nguyen phan 2
9 pt nghiem nguyen phan 29 pt nghiem nguyen phan 2
9 pt nghiem nguyen phan 2
Hồng Quang
 
11 hinh on tap htq
11 hinh on tap htq11 hinh on tap htq
11 hinh on tap htq
Hồng Quang
 
8 phuong trinh nghiem nguyen htq
8 phuong trinh nghiem nguyen htq8 phuong trinh nghiem nguyen htq
8 phuong trinh nghiem nguyen htq
Hồng Quang
 
9 [htq] de thi hsg
9 [htq] de thi hsg9 [htq] de thi hsg
9 [htq] de thi hsg
Hồng Quang
 
9 [htq] de thi hsg 2
9 [htq] de thi hsg 29 [htq] de thi hsg 2
9 [htq] de thi hsg 2
Hồng Quang
 
10 hpt bai giang lopluyenthi
10 hpt bai giang lopluyenthi10 hpt bai giang lopluyenthi
10 hpt bai giang lopluyenthi
Hồng Quang
 
10 ptvt lien hop lopluyenthi.vn
10 ptvt lien hop lopluyenthi.vn10 ptvt lien hop lopluyenthi.vn
10 ptvt lien hop lopluyenthi.vn
Hồng Quang
 
10 ptvt bien doi lopluyenthi.vn
10 ptvt bien doi lopluyenthi.vn10 ptvt bien doi lopluyenthi.vn
10 ptvt bien doi lopluyenthi.vn
Hồng Quang
 

More from Hồng Quang (20)

10 de tang hsg quan huyen thay hong tri quang
10 de tang hsg quan   huyen thay hong tri quang10 de tang hsg quan   huyen thay hong tri quang
10 de tang hsg quan huyen thay hong tri quang
 
Pt co ban bttl phan 6 ct0, ct1, ct2
Pt co ban bttl phan 6 ct0, ct1, ct2Pt co ban bttl phan 6 ct0, ct1, ct2
Pt co ban bttl phan 6 ct0, ct1, ct2
 
9 pp danh gia
9 pp danh gia9 pp danh gia
9 pp danh gia
 
9 can thuc nc
9 can thuc nc9 can thuc nc
9 can thuc nc
 
20 cach cm bdt nesbit
20 cach cm bdt nesbit20 cach cm bdt nesbit
20 cach cm bdt nesbit
 
9 drichle
9 drichle9 drichle
9 drichle
 
[Htq] toan 9 hsg tp hn dap an
[Htq] toan 9 hsg tp hn   dap an[Htq] toan 9 hsg tp hn   dap an
[Htq] toan 9 hsg tp hn dap an
 
9 [htq] de thi hsg 3 2 lopluyenthi
9 [htq] de thi hsg 3 2 lopluyenthi9 [htq] de thi hsg 3 2 lopluyenthi
9 [htq] de thi hsg 3 2 lopluyenthi
 
9 03 de thi tet 2016
9 03 de thi tet 20169 03 de thi tet 2016
9 03 de thi tet 2016
 
9 pt vo ti (co ban) htq
9 pt vo ti (co ban) htq9 pt vo ti (co ban) htq
9 pt vo ti (co ban) htq
 
Đồ thị hàm số - toán lớp 9
Đồ thị hàm số - toán lớp 9Đồ thị hàm số - toán lớp 9
Đồ thị hàm số - toán lớp 9
 
Ba dạng hệ phương trình cơ bản
Ba dạng hệ phương trình cơ bảnBa dạng hệ phương trình cơ bản
Ba dạng hệ phương trình cơ bản
 
9 pt nghiem nguyen phan 2
9 pt nghiem nguyen phan 29 pt nghiem nguyen phan 2
9 pt nghiem nguyen phan 2
 
11 hinh on tap htq
11 hinh on tap htq11 hinh on tap htq
11 hinh on tap htq
 
8 phuong trinh nghiem nguyen htq
8 phuong trinh nghiem nguyen htq8 phuong trinh nghiem nguyen htq
8 phuong trinh nghiem nguyen htq
 
9 [htq] de thi hsg
9 [htq] de thi hsg9 [htq] de thi hsg
9 [htq] de thi hsg
 
9 [htq] de thi hsg 2
9 [htq] de thi hsg 29 [htq] de thi hsg 2
9 [htq] de thi hsg 2
 
10 hpt bai giang lopluyenthi
10 hpt bai giang lopluyenthi10 hpt bai giang lopluyenthi
10 hpt bai giang lopluyenthi
 
10 ptvt lien hop lopluyenthi.vn
10 ptvt lien hop lopluyenthi.vn10 ptvt lien hop lopluyenthi.vn
10 ptvt lien hop lopluyenthi.vn
 
10 ptvt bien doi lopluyenthi.vn
10 ptvt bien doi lopluyenthi.vn10 ptvt bien doi lopluyenthi.vn
10 ptvt bien doi lopluyenthi.vn
 

Recently uploaded

GIÁO TRÌNH 2-TÀI LIỆU SỬA CHỮA BOARD MONO TỦ LẠNH MÁY GIẶT ĐIỀU HÒA.pdf
GIÁO TRÌNH 2-TÀI LIỆU SỬA CHỮA BOARD MONO TỦ LẠNH MÁY GIẶT ĐIỀU HÒA.pdfGIÁO TRÌNH 2-TÀI LIỆU SỬA CHỮA BOARD MONO TỦ LẠNH MÁY GIẶT ĐIỀU HÒA.pdf
GIÁO TRÌNH 2-TÀI LIỆU SỬA CHỮA BOARD MONO TỦ LẠNH MÁY GIẶT ĐIỀU HÒA.pdf
Điện Lạnh Bách Khoa Hà Nội
 
CHUYÊN ĐỀ BỒI DƯỠNG HỌC SINH GIỎI KHOA HỌC TỰ NHIÊN 9 CHƯƠNG TRÌNH MỚI - PHẦN...
CHUYÊN ĐỀ BỒI DƯỠNG HỌC SINH GIỎI KHOA HỌC TỰ NHIÊN 9 CHƯƠNG TRÌNH MỚI - PHẦN...CHUYÊN ĐỀ BỒI DƯỠNG HỌC SINH GIỎI KHOA HỌC TỰ NHIÊN 9 CHƯƠNG TRÌNH MỚI - PHẦN...
CHUYÊN ĐỀ BỒI DƯỠNG HỌC SINH GIỎI KHOA HỌC TỰ NHIÊN 9 CHƯƠNG TRÌNH MỚI - PHẦN...
Nguyen Thanh Tu Collection
 
AV6 - PIE CHART WRITING skill in english
AV6 - PIE CHART WRITING skill in englishAV6 - PIE CHART WRITING skill in english
AV6 - PIE CHART WRITING skill in english
Qucbo964093
 
Dẫn luận ngôn ngữ - Tu vung ngu nghia.pptx
Dẫn luận ngôn ngữ - Tu vung ngu nghia.pptxDẫn luận ngôn ngữ - Tu vung ngu nghia.pptx
Dẫn luận ngôn ngữ - Tu vung ngu nghia.pptx
nvlinhchi1612
 
Ảnh hưởng của nhân sinh quan Phật giáo đến đời sống tinh thần Việt Nam hiện nay
Ảnh hưởng của nhân sinh quan Phật giáo đến đời sống tinh thần Việt Nam hiện nayẢnh hưởng của nhân sinh quan Phật giáo đến đời sống tinh thần Việt Nam hiện nay
Ảnh hưởng của nhân sinh quan Phật giáo đến đời sống tinh thần Việt Nam hiện nay
chinhkt50
 
30 - ĐỀ THI HSG - HÓA HỌC 9 - NĂM HỌC 2021 - 2022.pdf
30 - ĐỀ THI HSG - HÓA HỌC 9 - NĂM HỌC 2021 - 2022.pdf30 - ĐỀ THI HSG - HÓA HỌC 9 - NĂM HỌC 2021 - 2022.pdf
30 - ĐỀ THI HSG - HÓA HỌC 9 - NĂM HỌC 2021 - 2022.pdf
ngocnguyensp1
 
Chương III (Nội dung vẽ sơ đồ tư duy chương 3)
Chương III (Nội dung vẽ sơ đồ tư duy chương 3)Chương III (Nội dung vẽ sơ đồ tư duy chương 3)
Chương III (Nội dung vẽ sơ đồ tư duy chương 3)
duykhoacao
 
98 BÀI LUYỆN NGHE TUYỂN SINH VÀO LỚP 10 TIẾNG ANH DẠNG TRẮC NGHIỆM 4 CÂU TRẢ ...
98 BÀI LUYỆN NGHE TUYỂN SINH VÀO LỚP 10 TIẾNG ANH DẠNG TRẮC NGHIỆM 4 CÂU TRẢ ...98 BÀI LUYỆN NGHE TUYỂN SINH VÀO LỚP 10 TIẾNG ANH DẠNG TRẮC NGHIỆM 4 CÂU TRẢ ...
98 BÀI LUYỆN NGHE TUYỂN SINH VÀO LỚP 10 TIẾNG ANH DẠNG TRẮC NGHIỆM 4 CÂU TRẢ ...
Nguyen Thanh Tu Collection
 
BAI TAP ON HE LOP 2 LEN 3 MON TIENG VIET.pdf
BAI TAP ON HE LOP 2 LEN 3 MON TIENG VIET.pdfBAI TAP ON HE LOP 2 LEN 3 MON TIENG VIET.pdf
BAI TAP ON HE LOP 2 LEN 3 MON TIENG VIET.pdf
phamthuhoai20102005
 
Khoá luận tốt nghiệp ngành Truyền thông đa phương tiện Xây dựng kế hoạch truy...
Khoá luận tốt nghiệp ngành Truyền thông đa phương tiện Xây dựng kế hoạch truy...Khoá luận tốt nghiệp ngành Truyền thông đa phương tiện Xây dựng kế hoạch truy...
Khoá luận tốt nghiệp ngành Truyền thông đa phương tiện Xây dựng kế hoạch truy...
https://www.facebook.com/garmentspace
 

Recently uploaded (10)

GIÁO TRÌNH 2-TÀI LIỆU SỬA CHỮA BOARD MONO TỦ LẠNH MÁY GIẶT ĐIỀU HÒA.pdf
GIÁO TRÌNH 2-TÀI LIỆU SỬA CHỮA BOARD MONO TỦ LẠNH MÁY GIẶT ĐIỀU HÒA.pdfGIÁO TRÌNH 2-TÀI LIỆU SỬA CHỮA BOARD MONO TỦ LẠNH MÁY GIẶT ĐIỀU HÒA.pdf
GIÁO TRÌNH 2-TÀI LIỆU SỬA CHỮA BOARD MONO TỦ LẠNH MÁY GIẶT ĐIỀU HÒA.pdf
 
CHUYÊN ĐỀ BỒI DƯỠNG HỌC SINH GIỎI KHOA HỌC TỰ NHIÊN 9 CHƯƠNG TRÌNH MỚI - PHẦN...
CHUYÊN ĐỀ BỒI DƯỠNG HỌC SINH GIỎI KHOA HỌC TỰ NHIÊN 9 CHƯƠNG TRÌNH MỚI - PHẦN...CHUYÊN ĐỀ BỒI DƯỠNG HỌC SINH GIỎI KHOA HỌC TỰ NHIÊN 9 CHƯƠNG TRÌNH MỚI - PHẦN...
CHUYÊN ĐỀ BỒI DƯỠNG HỌC SINH GIỎI KHOA HỌC TỰ NHIÊN 9 CHƯƠNG TRÌNH MỚI - PHẦN...
 
AV6 - PIE CHART WRITING skill in english
AV6 - PIE CHART WRITING skill in englishAV6 - PIE CHART WRITING skill in english
AV6 - PIE CHART WRITING skill in english
 
Dẫn luận ngôn ngữ - Tu vung ngu nghia.pptx
Dẫn luận ngôn ngữ - Tu vung ngu nghia.pptxDẫn luận ngôn ngữ - Tu vung ngu nghia.pptx
Dẫn luận ngôn ngữ - Tu vung ngu nghia.pptx
 
Ảnh hưởng của nhân sinh quan Phật giáo đến đời sống tinh thần Việt Nam hiện nay
Ảnh hưởng của nhân sinh quan Phật giáo đến đời sống tinh thần Việt Nam hiện nayẢnh hưởng của nhân sinh quan Phật giáo đến đời sống tinh thần Việt Nam hiện nay
Ảnh hưởng của nhân sinh quan Phật giáo đến đời sống tinh thần Việt Nam hiện nay
 
30 - ĐỀ THI HSG - HÓA HỌC 9 - NĂM HỌC 2021 - 2022.pdf
30 - ĐỀ THI HSG - HÓA HỌC 9 - NĂM HỌC 2021 - 2022.pdf30 - ĐỀ THI HSG - HÓA HỌC 9 - NĂM HỌC 2021 - 2022.pdf
30 - ĐỀ THI HSG - HÓA HỌC 9 - NĂM HỌC 2021 - 2022.pdf
 
Chương III (Nội dung vẽ sơ đồ tư duy chương 3)
Chương III (Nội dung vẽ sơ đồ tư duy chương 3)Chương III (Nội dung vẽ sơ đồ tư duy chương 3)
Chương III (Nội dung vẽ sơ đồ tư duy chương 3)
 
98 BÀI LUYỆN NGHE TUYỂN SINH VÀO LỚP 10 TIẾNG ANH DẠNG TRẮC NGHIỆM 4 CÂU TRẢ ...
98 BÀI LUYỆN NGHE TUYỂN SINH VÀO LỚP 10 TIẾNG ANH DẠNG TRẮC NGHIỆM 4 CÂU TRẢ ...98 BÀI LUYỆN NGHE TUYỂN SINH VÀO LỚP 10 TIẾNG ANH DẠNG TRẮC NGHIỆM 4 CÂU TRẢ ...
98 BÀI LUYỆN NGHE TUYỂN SINH VÀO LỚP 10 TIẾNG ANH DẠNG TRẮC NGHIỆM 4 CÂU TRẢ ...
 
BAI TAP ON HE LOP 2 LEN 3 MON TIENG VIET.pdf
BAI TAP ON HE LOP 2 LEN 3 MON TIENG VIET.pdfBAI TAP ON HE LOP 2 LEN 3 MON TIENG VIET.pdf
BAI TAP ON HE LOP 2 LEN 3 MON TIENG VIET.pdf
 
Khoá luận tốt nghiệp ngành Truyền thông đa phương tiện Xây dựng kế hoạch truy...
Khoá luận tốt nghiệp ngành Truyền thông đa phương tiện Xây dựng kế hoạch truy...Khoá luận tốt nghiệp ngành Truyền thông đa phương tiện Xây dựng kế hoạch truy...
Khoá luận tốt nghiệp ngành Truyền thông đa phương tiện Xây dựng kế hoạch truy...
 

9 tu giac noi tiep

  • 1. Luyện thi học sinh giỏi, thi chuyên toán lớp 10 Thầy Hồng Trí Quang 1 Nội dung Phần 1. Kiến thức bổ sung 1.Số đo cung – góc ở tâm – góc nội tiếp 2.Góc ở trong và ngoài đường tròn 3.Góc tạo bởi tiếp tuyến và dây cung Phần 2. Chứng minh tứ giác nội tiếp Dấu hiệu 1. Tổng hai góc đối bằng 1800 Dấu hiệu 2. Hai góc cùng nhìn một cung bằng nhau Dấu hiệu 3. Dấu hiệu tích. Phần 3. Ứng dụng tứ giác nội tiếp Ứng dụng hai hệ thức Định lí P tô lê mê Chứng minh ba điểm thẳng hàng Chứng minh ba đường đồng quy. Tài liệu này của khóa học “Luyện thi học sinh giỏi, thi chuyên toán lớp 10” của thầy Hồng Trí Quang Facebook thảo luận https://www.facebook.com/chuyentoanlop9/?ref=bookmarks SỐ ĐO CUNG – GÓC Ở TÂM – GÓC NỘI TIẾP Bài 1. Cho tam giác ABC nội tiếp (O). Tia phân giác góc A cắt BC tại F, cắt (O) tại E. Chứng minh: 1/ Tam giác BEC cân 2/ BEC ABC ACB  3/ . .AB AC AE AF 4/ 2 . .AF AB AC BF CE  HD: Bài đơn giản, câu 3, 4 sử dụng tam giác đồng dạng. Bài 2. Cho đường tròn (O) tâm O, đường kính AB = 2R. Gọi PQ là một dây thay đổi của đường tròn (O) sao cho PQ = R. Vẽ hình bình hành PAQM.
  • 2. Luyện thi học sinh giỏi, thi chuyên toán lớp 10 Thầy Hồng Trí Quang 2 a) Chứng minh rằng B là trực tâm tam giác MPQ b) Tính theo R khoảng cách từ tâm O đến PQ c) Khi dây PQ thay đổi thì điểm M di động trên đường nào? HD c) gọi H là giao AM và PQ, thì OH là đường trung bình. Điểm M di động trên đường tròn tâm B bán kính Rsqrt3 Bài 3. Điểm M tùy ý trên đoạn AB cố định. Trên AM và MB dựng về một phía đối với AB hai hình vuông. Các đường tròn ngoại tiếp các hình vuông cắt nhau tại điểm thứ hai N. a) Chứng minh rằng đường thẳng AN đi qua một đỉnh của hình vuông thứ hai b) Tìm quỹ tích của điểm N khi điểm M di động trên AB c) Tìm quỹ tích trung điểm I của đoạn thẳng nối hai tâm hình vuông HD a) 0 0 0 45 45 90ANB ANM NMB     , tương tự: 0 90DNB  ; b) N nằm trên đường tròn đường kính AB; giới hạn trong nửa đường tròn; c) Từ hai tâm hạ hai đường vuông góc; khoảng cách từ I đến AB bằng AB/4, giới hạn khi M trùng A, M trùng B là đoạn PQ (P, Q cũng chính là giao của đt cách AB một đoạn AB/4 cắt AE và BD). Luyện tập Góc … Bài 4. Cho tam giác ABC nội tiếp đường tròn. Lấy điểm D trên cung BC (không chứa A) của đường tròn đó. Vẽ DH vuông góc với BC; DI vuông góc với CA và BK vuông góc với AB. Chứng minh rằng: BC AC AB DH DI DK   M P Q A B
  • 3. Luyện thi học sinh giỏi, thi chuyên toán lớp 10 Thầy Hồng Trí Quang 3 HD ý tưởng của bài toán là tìm điểm M trên BC để tách tỉ số: BC BM MC DH DH DH   , và từng tỉ số của tổng này ứng với tỉ số cần chứng minh. Trên cạnh BC lấy điểm M sao cho BMD ADC ( . )BMD ADC g g  DH BM DI AC   (tỉ số đường cao bằng tỉ số đồng dạng) AC BM DI DH   Chứng minh tương tự thì: AB CM DK DH  Cộng lại ta có Đpcm Bài 5. Tam giác ABC vuông tại A, nội tiếp đường tròn (O) đường kính 4 2cm. Tiếp tuyến tại C của đường tròn cắt tia phân giác góc B tại K. Tính độ dài BK biết BK cắt AC tại D và BD = 4cm. Hướng dẫn Gọi giao điểm hai đường tròn là M. Đặt MD = MK = x. Từ 2 .BC BM BK ; 2 17 2BK   (cm) Bài 6. Từ điểm A nằm ngoài đường tròn (O) kẻ hai tiếp tuyến AB, AC (B, C là tiếp điểm). Gọi M là trung điểm AC. Đoạn thẳng MB cắt đường tròn tại K (khác B). Tia AK cắt đường tròn tại D (khác K). Chứng minh rằng BD song song với AC HD Ta có: MKC CBM MCK MBC   MC MK MB MC   MC MK MB MA   (vì MA = MC) Vì AMK chung nên MAK MBA  MAK ABM  Mà BDK ABM nên BDK KAM Đpcm Bài 7. Cho hai đường tròn (O; R) và (O’, r) tiếp xúc trong tại A (R > r). Dây BC của đường tròn (O;R) tiếp xúc với (O’; r) tại M. Chứng minh rằng AM là tia phân giác góc BAC HD Kẻ tiếp tuyến chung từ A cắt BC tại F. Vì: MBA CAF , mà FMA FAM suy ra Đpcm Bài 8. Cho hai đường tròn (O) và (O’) tiếp xúc ngoài tại I, AB là tiếp tuyến chung. Kẻ đường kính AOD, từ D kẻ tiếp tuyến DE với đường tròn (O’). Chứng minh tam giác DAE cân.
  • 4. Luyện thi học sinh giỏi, thi chuyên toán lớp 10 Thầy Hồng Trí Quang 4 HD a) Kẻ tiếp tuyến chung tại I; b) Kẻ tiếp tuyến chung AB, chứng minh D, I, B thẳng hàng;  2 2 .DB DDE DI A Bài 9. Từ điểm M nằm ngoài đường tròn (O), vẽ hai tiếp tuyến MA, MB và cát tuyến MCD với đường tròn (O). Gọi I là giao điểm của AB và CD. Chứng minh rằng: IC MC ID MD  HD: Khi chứng minh các đẳng thức tích, người ta thường vận dụng các tam giác đồng dạng. Nhưng nếu không ghép được trực tiếp vào các tam giác đồng dạng thì nên vận dụng thông qua các đẳng thức trung gian. Từ các cặp tam giác đồng dạng ta có: ( . )IAC IDB g g  AC IC DB IB   ( . )IBC IDA g g  BC IB AD ID   Suy ra: . . IC IC IB AC BC ID IB ID DB DA   Từ đó ta chứng minh: . MC AC BC MD DB DA  Xét các cặp tam giác đồng dạng ( . )MAC MDA g g  AM MC AC DM MA DA    AM AC DM DA   và 2 .MA MC MD ( . )MBC MDB g g  BM BC DM DB   Ta có: 2 .MC MC MD MD MD  2 2 MA MD  . MA MA MD MD  AC BC AD BD  Bài 10.*Cho hai đường tròn (O) và (O’) cắt nhau tại A và B. Các điểm M, N theo thứ tự di chuyển trên (O), (O’) sao cho chiều đi từ A đến M và từ A đến N trên các đường tròn theo chiều kim đồng hồ và sđ AM = sđ AN . Chứng minh rằng đường trung trực MN luôn đi qua một điểm cố định. Hướng dẫn Vẽ hình dự đoán điểm cố định nằm trên đường thẳng đi qua A, vuông góc AB Qua A kẻ đường thẳng vuông góc với AB cắt (O) và (O’) lần lượt tại C, D. Gọi I là trung điểm CD thì I cố định. Ta chứng minh I nằm trên trung trực MN.
  • 5. Luyện thi học sinh giỏi, thi chuyên toán lớp 10 Thầy Hồng Trí Quang 5 Gọi H là trung điểm MN thì IH là đường trung bình của hình thang vuông MDNC. Đpcm. Bài 11.*Cho hai đường tròn (O; R) và (O’, R’) cắt nhau tại hai điểm A, B. Tiếp tuyến với (O) tại A cắt (O’) tại C. Tiếp tuyến với (O’) tại A cắt (O) tại D. Gọi M là giao điểm hai đường thẳng AB và CD. Gọi N là trung điểm của CD. Chứng minh rằng: 1/ 2 2 'MC R MD R  2/ CAM DAN Hướng dẫn 1/ BAC BDA  'BA BC AC R BD BA DA R     và BM là phân giác góc DBC Từ đó: 2 2 'MC BC BC BA R MD BD BA BD R    2/ Gọi I là trung điểm AD, cộng trừ góc ta có: AIN ABC . Từ đó: ( . . )AIN ABC c g c  . Đpcm Bài 12. *Cho đường tròn (O), từ điểm M nằm ngoài đường tròn kẻ hai tiếp tuyến MA và MB. Đường tròn đi qua M và tiếp xúc với AB tại B cắt đường tròn (O) tại C. Chứng minh rằng AC đi qua trung điểm MB. Dự đoán h.b.h, tạo hình bình hành HD Gọi D là giao điểm của AC với đường tròn qua M tiếp xúc AB. ABDM là h.b.h do DM C CAB Bài 13.*Cho tam giác ABC nội tiếp đường tròn (O), A là điểm cố định; B và C thay đổi. Kẻ đường cao BH và CK. Chứng minh rằng HK luôn song song với một đường thẳng cố định. HD Kẻ tiếp tuyến Ax, chứng minh AKH KAx . Chú ý: chứng minh AKH HCB vì cùng phụ với góc HBC Bài 14. *Cho đường tròn (O), từ điểm M nằm ngoài đường tròn kẻ hai tiếp tuyến MA, MB, gọi D là điểm trên cung lớn AB , đường thẳng AD cắt MB kéo dài tại E sao cho D là trung điểm của AE. Đường thẳng MD cắt đường tròn (O) tại I. Chứng minh rằng BI song song với AD. GÓC TRONG VÀ GÓC NGOÀI ĐƯỜNG TRÒN Chú ý: Hai dây cung song song tạo ra hai cung có số đo bằng nhau.
  • 6. Luyện thi học sinh giỏi, thi chuyên toán lớp 10 Thầy Hồng Trí Quang 6 Bài 15. Cho đường tròn (O; R) với ba dây liên tiếp AB, BC, CD bằng nhau và cùng nhỏ hơn R. Các đường thẳng AB, CD cắt nhau tại I, các tiếp tuyến của đường tròn tại B và D cắt nhau tại K. Chứng minh rằng: 1/ BIC BKD 2/ IBC KBD  ; CBD IBK  Hướng dẫn 2/ TH1 (g.g), đồng dạng thứ 2 (c.g.c) HD cộng trừ góc – bài dễ Bài 16.Cho tam giác ABC (AC < AB) nội tiếp trong đường tròn tâm O bán kính R. Đường phân giác trong và ngoài của góc A cắt đường thẳng BC theo thứ tự tại D, E sao cho AD = AE. Tính 2 2 AB AC theo R. Hướng dẫn: Đưa hai cạnh AB, AC về độ dài hai cạnh của tam giác vuông. Gọi AD cắt (O) tại M, kẻ đường kính BF. Vì: FA AC nên AF = AC. Do đó: 2 2 2 4AB AC R  Bài 17. Cho tam giác ABC có AD là đường phân giác. Vẽ đường tròn tâm (O) đi qua A và D đồng thời tiếp xúc với BC tại D. Đường tròn này cắt AB, AC lần lượt tại E và F. Chứng minh rằng: a) EF song song với BC b) AED ADC  và AFD ADB  c) 2 . .AE AC AF AB AD  Bài 18. Cho đường tròn (O; R) với ba dây cung liên tiếp AB, BC, CD bằng nhau và cùng nhỏ hơn R. Các đường thẳng AB, CD cắt nhau tại I. Các tiếp tuyến của đường tròn tại B và D cắt nhau tại K. Chứng minh rằng a) BIC BKD b) IBC KBD  c) CBD IBK  Bài 19.Từ điểm E ở bên ngoài đường tròn (O) kẻ hai cát tuyến EAB, EDC sao cho AB < CD. Tia DA và CB cắt nhau tại F. Tia phân giác của hai góc CEB và CFD cắt nhau tại I. Chứng minh rằng EI FI Hướng dẫn Để chứng minh 0 90FIE  ta chứng minh: PN MQ NQ MP   PP CHỨNG MINH TỨ GIÁC NỘI TIẾP VÀ ỨNG DỤNG
  • 7. Luyện thi học sinh giỏi, thi chuyên toán lớp 10 Thầy Hồng Trí Quang 7 Dấu hiệu 1 Chỉ ra hai đỉnh kề nhau cùng nhìn một cung một góc bằng nhau (ví dụ D DCA CB ) Dấu hiệu 2: Chỉ ra tổng hai góc đối nhau bằng 1800 . Dấu hiệu 3: Mở rộng Nếu tứ giác ABCD có AB cắt CD tại M thỏa mãn MA.MB = MC.MD thì ABCD nội tiếp Nếu AC cắt BD tại N thỏa mãn NA.NC = NB.ND thì ABCD nội tiếp. Chú ý. 1. Điểm M nằm ngoài đường tròn (O). Nếu qua M kẻ tiếp tuyến MN và hai cát tuyến MAB, MCD thì 2 . .MAMB MC MD MN  2. Chứng minh năm điểm A, B, C, D, E thuộc cùng một đường tròn, +) Ta chứng minh 2 tứ giác bất kì (ví dụ ABCD, ABCE) nội tiếp. +) Ta chứng minh 5 điểm cách đều một điểm cho trước Dấu hiệu 1. Tổng hai góc đối bằng 1800 Bài 20. Từ 1 điểm M ở ngoài (O), vẽ 2 tiếp tuyến MA, MB với đường tròn. Trên cung nhỏ AB lấy 1 điểm C. Vẽ CD vuông góc với AB, CE vuông góc với MA, CF vuông góc với MB. Gọi I là giao điểm của AC và DE, K là giao điểm của BC và DF. Chứng minh rằng a) Tứ giác ICKD nội tiếp b) IK vuông góc với CD c) CD2 = CE.CF (góc tiếp tuyến và dây cung – hai tam giác đồng dạng) a) ta có: 1 1A B (cùng chắn cung AC) + do tứ giác BFCD nt 1 1F B (cùng chắn cung CD) C 2 2 2 2 2 1 1 1 1 1 K I F E D O B M A
  • 8. Luyện thi học sinh giỏi, thi chuyên toán lớp 10 Thầy Hồng Trí Quang 8 Suy ra: 1 1F A (1) + do tứ giác AECD nt 1 1A D (cùng chắn cung CE) (2) Từ (1) và (2) suy ra: 1 1 1F D B  Mặt khác: 2 2A B (cùng chắn cung BC) + do tứ giác AECD nt 2 2A E (cùng chắn cung CD) Suy ra: 2 2E B (3) + do tứ giác BFCD nt 2 2D B (cùng chắn cung CF) (4) Từ (3) và (4) suy ra: 2 2 2E D A  b) Xét tứ giác ICKD, ta có: 0 1 2 1 2 180ICK IDK ICK D D ACB B A        (tổng các góc của tam giác ABC), mà ;ICK IDK là 2 góc ở vị trí đối nhau, suy ra tứ giác ICKD nt d) ta có tứ giác ICKD nt 1 2I D (cùng chắn cung CK), mà 2 2D A (cmt) Suy ra 1 2I A , mà 1 2;I A là 2 góc ở vị trí đồng vị nên IK // AB, lại do AB vuông góc với CD, nên IK vuông góc với CD c) Xét tam giác CDE và tam giác CDF, ta có:  1 1 2 2 2 . . D F CD CE CDE CFD g g CD CE CF CF CDE D            Bài 21.Cho đường tròn tâm O đường kính AB. Trên một nửa đường tròn đường kính AB lấy điểm C, D sao cho AC AD (D khác B). Trên nửa đường tròn còn lại lấy điểm E (khác A và B). CE cắt AD tại I. Đường thẳng IO cắt BE tại K. a) Gọi F là điểm đối xứng của D qua IK, chứng minh tứ giác IFEK nội tiếp b) Chứng minh tam giác CDK vuông. Hướng dẫn. Bỏ a a) BIK DIK FDB FAB   Mà 0 180FAB FEB  nên tứ giác IFEK nội tiếp.
  • 9. Luyện thi học sinh giỏi, thi chuyên toán lớp 10 Thầy Hồng Trí Quang 9 b) Ta có IEK IFK IDK  Vậy 0 180IDK CDB  0 0 90 180IDK CDI    . Đpcm. Tự luyện Bài 22.Cho tam giác nhọn ABC (AB < AC). Các đường cao BD, CE cắt nhau tại H. Gọi M là giao điểm của BC và DE. Gọi O là trung điểm cạnh BC. Gọi P (khác O) là giao điểm của các đường tròn ngoại tiếp các tam giác OBE và OCD. Chứng minh rằng: a) Tứ giác HEPD nội tiếp b) Tứ giác MEPC nội tiếp c) Chứng minh MP AO Hướng dẫn a; b bỏ a) Chứng minh A, D, P, E, H cùng nằm trên đường tròn. b) Chú ý. O là tâm ngoại tiếp BEDC, và sử dụng a chứng minh b.    DEC ODB PED PDO PCO c) MPC MEC và   MEB OCD ODC OPC nên MP PO Tuy nhiên, cần chứng minh A, P, O thẳng hàng. Do 0 180   APD DPO OCD DPO Dấu hiệu 2. Hai góc cùng nhìn một cung Bài 23.Cho hai đường tròn (O) và (O’) cắt nhau tại A và B. Đường thẳng d cắt (O) và (O’) lần lượt tại C và D. Hai tia CO và DO’ cắt nhau ở E. Chứng minh rằng C, A, D, E cùng thuộc một đường tròn. HD 'D  AOC AO ACO ADE Bài 24.Cho tam giác ABC, điểm M di động trên cạnh BC. Các đường trung trực của các đoạn thẳng BM, CM lần lượt cắt AB, AC tại D, E. a) Gọi S là điểm đối xứng của M qua DE, SM cắt AH tại K, chứng minh tứ giác SAKB nội tiếp b) Chứng minh rằng đường thẳng qua M và vuông góc với DE luôn đi qua điểm cố định. Hướng dẫn. Bỏ a a) Chứng minh 1 2 BSK BDM ABK 
  • 10. Luyện thi học sinh giỏi, thi chuyên toán lớp 10 Thầy Hồng Trí Quang 10 b) Đã có BSAK nội tiếp, nhưng chưa đủ suy ra K cố định. Ta cần chứng minh S, K thuộc đường tròn ngoại tiếp tam giác ABC, tức chứng minh SACK nội tiếp (chứng minh tương tự). Tự luyện Bài 25.Cho tam giác ABC nội tiếp (O) có 2AB < BC. Trên cạnh BC lấy điểm D sao cho CD = AB. Đường thẳng qua D song song với AC cắt AB ở E. Tia phân giác góc ABC cắt DE ở F. a) Gọi N là giao điểm của AF và đường tròn (O. Đường thẳng qua D song song với AB cắt AF ở M. Chứng minh tứ giác DNMC nội tiếp. b) Chứng minh rằng đường thẳng AF đi qua trung điểm I của BD. Hướng dẫn. Bỏ a a) Ta có NMD BAN NCD  b) Để chứng minh I là trung điểm BD ta chứng minh tứ giác ADMB là hình bình hành, tức chứng minh hai cặp cạnh đối AB và DM song song và bằng nhau. 2 2 2BDM ABD FBD FND DCM    Mà BDM ABD DCM DMC   nên DM = DC = AB. Bài 26.Cho đường tròn (O) đường kính AB. Trên tiếp tuyến của đường tròn (O) tại A lấy điểm M, vẽ cát tuyến MCD (C nằm giữa M và D). Gọi E, F lần lượt là giao điểm của OM với BC và BD. a) Kẻ tiếp tuyến MN, chứng minh tứ giác MCNE nội tiếp. b) Chứng minh rằng OE = OF. Hướng dẫn. Bỏ a a) Chứng minh NME NAO NCE  b) AOE BOF   EAO OBF  MAE OAD  MNE BCD  (Theo a) Cách 2. Qua C kẻ song song với EF cắt AB, BD tại H, K. Kẻ ON vuông CD tại N, chứng minh ACHN nội tiếp và NH song song BD. Suy ra H là trung điểm CK. Sử dụng talet ta chứng minh được OE BO OF CH BH HK   Bài 27.*Cho tam giác nhọn ABC. Trên nửa mặt phẳng bờ AB không chứa C vẽ tia Ax, trên nửa mặt phẳng bờ AC không chứa B vẽ tia Ay sao cho 0 90yAC xAB  . Vẽ BD Ax tại D, CE Ay tại E. Vẽ AH là đường cao của tam giác ABC. Gọi M là trung điểm BC. Chứng minh rằng M, H, D, E cùng thuộc một đường tròn. Hướng dẫn Chứng minh hai góc cùng nhìn cung bằng nhau
  • 11. Luyện thi học sinh giỏi, thi chuyên toán lớp 10 Thầy Hồng Trí Quang 11 Ta có 0 180DHE DHB EHC   0 180 2DAB  Để sử dụng giả thiết M là trung điểm BC, ta lấy thêm I, K lần lượt là trung điểm AB, AC. Khi đó ( . . )AIM MKE c g c   DMH AMI IMK KME    IDM DMI BAC    0 0 180 180DIM BAC DIB     0 180 2DAB  DHE Tam giác bằng nhau, tam giác đồng dạng. Bài 28.Cho nửa đường tròn tâm O, đường kính AB, M là điểm đối xứng của O qua A. Đường thẳng qua M cắt nửa đường tròn (O) tại C và D (C nằm giữa M và D). Gọi E là giao điểm của AD và BC. a) Gọi N là trung điểm của OA, chứng minh tứ giác NEDB nội tiếp b) Chứng minh rằng 3 BC AE AD BE  HD Bỏ a. a) Ta chứng minh NDE NBE Có NBE ADC Nên NDE NBE NDA ADC  DMA NDO  DMO NDO   (c.g.c) b) Áp dụng tính chất của tứ giác nội tiếp ta có Bài 29.*Cho tam giác ABC vuông tại A có AM là đường trung tuyến, AH là đường cao. Trên tia đối của tia AM lấy điểm P (P khác A). Các đường thẳng qua H vuông góc với AB và AC lần lượt cắt các đường thẳng PB và PC tại Q và R tương ứng. a) Gọi E là giao điểm của AC và PB, F là giao điểm của AB và PC. Qua P kẻ đường thẳng song song với BC cắt AC, AB lần lượt tại E’, F’. Gọi I là giao điểm của HQ và AB, K là giao điểm của HR và AC. Chứng minh IK song song với QR. b) Chứng minh tứ giác BHAS nội tiếp với S là giao điểm của RA và PB c) Chứng minh A là trực tâm tam giác PQR Hướng dẫn. Bỏ a; b 3 . 3 . . .  AE AE AN AB BN BA BE BC
  • 12. Luyện thi học sinh giỏi, thi chuyên toán lớp 10 Thầy Hồng Trí Quang 12 a) Talet, chứng minh P là trung điểm E’F’ Talet, chứng minh EF song song BC. Talet, chứng minh IK song song với QR b) ;   HQ HI HB BHQ AHR HR HK HA (c. .c)  BHQ AHE g nên BHAS nội tiếp c) SR vuông PQ, PA vuông QR nên A là trực tâm. Tự luyện Bài 30. Nếu hai đường thẳng AB, CD cắt nhau tại điểm M và . .MAMB MC MDthì bốn điểm A, B, C, D thuộc cùng một đường tròn. Bài 31.Cho đường tròn (O), điểm K nằm ngoài đường tròn. Kẻ các tiếp tuyến KA, KB và cát tuyến KCD với đường tròn. Gọi M là giao điểm của OK và AB. Kẻ OH vuông góc với CD, cắt AB ở E. Chứng minh rằng: a) CMOE là tứ giác nội tiếp b) CE, DE là các tiếp tuyến của (O). HD cm góc CME = COE Bài 32. Cho tam giác ABC có 3 góc nhọn. Đường tròn tâm O đường kính BC cắt AB tại E, cắt AC tại F. Các tia BE cà CE cắt nhau tại H. Gọi M là trung điểm của BH. Chứng minh rằng tứ giác EMKF Bài 33. Cho tam giác ABC. Các điểm D, E di động trên các tia BA, CA sao cho 3BD = 2CE. a) Vẽ (O’) ngoại tiếp tam giác ABC. Gọi M là điểm chính giữa cung BC (M và A nằm khác phía đối với BC). I là điểm trên cạnh BC và 3 2BI IC , MI cắt (O’) tại N khác M. a) Chứng minh tứ giác ADEN nội tiếp. b) Chứng minh rằng tâm O của đường tròn ngoại tiếp tam giác ADE thuộc một đường thẳng cố định. Hướng dẫn. Bỏ a a) NB IB BD NC IC CE   nên NBD NCE  . Đpcm b) Tứ giác ADEN nội tiếp nên OA = ON, tức O thuộc đường thẳng cố định là trung trực của AN Bài 34.*Cho tam giác ABC nhọn, ba đường cao AD, BE và CF cắt nhau tại H. Qua A vẽ các đường thẳng song song với BE, CF lần lượt cắt các đường thẳng CF, BE tại P và Q. a) Chứng minh tứ giác AKEQ nội tiếp. b) Chứng minh rằng PQ vuông góc với trung tuyến AM của tam giác ABC. Hướng dẫn. Bỏ a
  • 13. Luyện thi học sinh giỏi, thi chuyên toán lớp 10 Thầy Hồng Trí Quang 13 a) Gọi I là giao điểm của AH và PQ, K là giao điểm của AM và PQ. Ta có I là trung điểm AH. MAC IQH AMC QIH   ; AC MC ACM IHQ QH IH    ACM IHQ (Hai góc nhọn có cạnh tương ứng vuông góc) AC MC QH IH  AC BC QH AH   ( . )ABC QAH g g   (hai góc có cạnh tương ứng vuông góc) b) Hiển nhiên Bài 35.*Cho hình thoi ABCD có 0 60BAD  . Đường thẳng d qua C cắt AB, AD lần lượt ở M và N. Gọi K là giao điểm của BN và DM. Chứng minh rằng A, B, D, K cùng thuộc một đường tròn. Hướng dẫn Từ A kẻ đường thẳng song song với BD, E thuộc BN MA NA AE CD ND BD   MA AE  ( . . )AEB AMD c g c    . Đpcm. DẤU HIỆU 4. SỬ DỤNG DẤU HIỆU TÍCH ĐỂ CHỨNG MINH TỨ GIÁC NỘI TIẾP Trong chương trình toán nâng cao hình học lớp 9, chứng minh tứ giác và sử dụng tứ giác nội tiếp là rất quan trọng. Trong đó dấu hiệu tích là dấu hiệu để nhận biết và chứng minh nhanh tứ giác nội tiếp mà các bạn chưa khai thác. Một vài ví dụ dưới đây sẽ chỉ cho các bạn thấy ứng dụng đơn giản mà tuyệt vời này. Tính chất tích Thuận. Nếu hai đường thẳng chứa các dây AB, CD của một đường tròn cắt nhau tại một điểm M (nằm trong hoặc nằm ngoài đường tròn) thì: . .MAMB MC MD Đảo. Nếu hai đường thẳng AB, CD cắt nhau tại điểm M và . .MAMB MC MDthì bốn điểm A, B, C, D thuộc cùng một đường tròn. Hướng dẫn. Sử dụng tam giác đồng dạng để chứng minh. Chú ý. Nếu M nằm ngoài đường tròn (O), cát tuyến MAB và tiếp tuyến MP thì 2 .MAMB MP Bài 36.Từ điểm A nằm ngoài đường tròn (O) vẽ các tiếp tuyến AM, AN đến đường tròn (O) và cát tuyến ADC. B là điểm trên cung CM (không chứa D). Gọi H là giao điểm của MN và BD, E là giao điểm của CH và đường tròn (O).
  • 14. Luyện thi học sinh giỏi, thi chuyên toán lớp 10 Thầy Hồng Trí Quang 14 a) Gọi K là giao điểm của AH và đường tròn ngoại tiếp tam giác (AMN). Chứng minh tứ giác AEKC và HKCD nội tiếp. b) Chứng minh rằng A, E, B thẳng hàng. Hướng dẫn. Bỏ a a) Xét . . .HKHE HC HM HN HA  2 . . .AH AK AF AO AN AC AD   b) 0 180AEC BEC HKC EDC    . Đpcm Bài 37.Cho điểm A nằm ngoài đường tròn (O). Vẽ AB, AC là các tiếp tuyến và cát tuyến ADE (tia AD nằm giữa hai tia AB và AO). OA cắt BC tại H. Vẽ BK vuông góc với DE tại K, KH cắt AB tại G và cắt đường thẳng đi qua A song song với CD tại M. Vẽ AS vuông góc với GD tại S. Chứng minh tứ giác DKMS nội tiếp Hướng dẫn Hạ DT vuông góc với AB tại T Ta có . .GT GA GDGS Nên tứ giác DKMS nội tiếp . .GT GA GK GM   tứ giác ATKM nội tiếp BTK AMK  Ta có BTK TKA TAK VCH VHC DVH AMK      Bài 38. Cho hình thang ABCD (AB//CD, AB < CD) có đường tròn (O) qua A, D tiếp xúc với BC ở N, đường tròn (I) đi qua B, C tiếp xúc với AD ở M. Chứng minh rằng tứ giác MNCD nội tiếp Hướng dẫn Gọi S là giao điểm của AD và BC Xét tích ta có 2 .SM SB SC ; 2 .SN SASD Cần chứng minh . .SM SD SN SC 2 2 2 2 SM SC SN SD   2 2 . . SB SC SC SA SD SD   SB SC SA SD   (luôn đúng) I H O G A B F N K H E I N C
  • 15. Luyện thi học sinh giỏi, thi chuyên toán lớp 10 Thầy Hồng Trí Quang 15 Bài 39.Cho đường tròn (O) có dây cung BC (khác đường kính) cố định, A là điểm chuyển động trên cung lớn BC, M là trung điểm dây BC. Gọi D là giao điểm của AM và cung nhỏ BC, N là giao điểm của AB và CD. a) Gọi E là giao điểm của các tiếp tuyến của đường tròn (O) tại B và C. Chứng minh tứ giác AODE nội tiếp, tứ giác BNED nội tiếp. b) Chứng minh rằng N thuộc một đường thẳng cố định. Hướng dẫn. Bỏ a a) Dễ thấy MA.MD = MB.MC = MD.MO Để chứng minh tứ giác NEDB nội tiếp, ta chứng minh hai góc bằng nhau NDE NBE EDC EBA  EKB EBA  AEO DEO  OA OD  (luôn đúng) b) END NBD END BCD  / /EN BC Vậy N thuộc đường thẳng cố định, đi qua E cố định và song song với BC. Bài 40. Cho tứ giác BFEC nội tiếp đường tròn đường kính BC (BF < CE). BE cắt CF tại H, FD cắt đường tròn (O) tại M, ED cắt đường tròn (I) tại N. Chứng minh tứ giác DMSN nội tiếp Hướng dẫn Gọi A là giao điểm BF và CE, ta có A, H, D thẳng hàng ; . . .FD FM FH FC FAFB  ; . . .EEN ED EH EB EA C  . : . . FM FA FB FD FA FB ED EN EA EC ED EA FD EC   . . 1 AC BC AB AB AC BC   FM EN  Vậy (c.g.c)  SFM SEN . Đpcm. Bài 41.*(Thi hsg cấp tỉnh – đề số 3) Cho hai đường tròn (O1) và (O2) tiếp xúc ngoài nhau tại điểm T. Hai đường tròn này nằm trong đường tròn (O3) và tiếp xúc với (O3) tương ứng tại M và N. Tiếp tuyến chung tại T của (O1) và (O2) cắt (O3) tại P. PM cắt đường tròn (O1) tại điểm thứ hai A và MN cắt (O1) tại điểm thứ hai B. PN cắt đường tròn (O2) tại điểm D và MN cắt (O2) tại điểm thứ hai C. a) Chứng minh rằng tứ giác AMND là tứ giác nội tiếp b) Chứng minh AB song song với PN. c) Gọi E là giao điểm của AB và CD, chứng minh . .EB PN EC PM b) Chứng minh rằng tứ giác ABCD nội tiếp.
  • 16. Luyện thi học sinh giỏi, thi chuyên toán lớp 10 Thầy Hồng Trí Quang 16 HD a) Theo dấu hiệu ta có 2 . .PA PM PT PD PN  nên tứ giác AMDN nội tiếp b) 1 1 3 3 O A O BMA MB MP O P O N MN    / /AB NP c) EBC PNM  d) EB PM PA EA EC PN PD ED    Luyện tập Bài 42.Cho tam giác nhọn ABC. Vẽ đường tròn (O) đường kính BC. Vẽ AM là tiếp tuyến của (O). Kẻ MK vuông góc với OA tại K. Gọi H là trực tâm của tam giác ABC, D là giao điểm của AH và BC. Chứng minh rằng D, H, K, O cùng thuộc một đường tròn. Hướng dẫn. Xét tích 2 . . .AH AD AN AB AM AK AO   Bài 43.Cho đường tròn (O), điểm K nằm ngoài đường tròn. Kẻ các tiếp tuyến KA, KB và cát tuyến KCD với đường tròn. Gọi M là giao điểm của OK và AB. Chứng minh rằng: a) CMOD là tứ giác nội tiếp b) Đường thẳng AB chứa tia phân giác của góc CMD. Hướng dẫn C M O K A B D  E N M B C O1 O3 O2 D P A T
  • 17. Luyện thi học sinh giỏi, thi chuyên toán lớp 10 Thầy Hồng Trí Quang 17 a) 2 . . MAMO MA MC MD nên tứ giác OMCD nội tiếp b) Để chứng minh CMB DMB ta chứng minh CMK DMO Thật vậy   CMK ODC OCD DMO Bài 44.Cho tam giác ABC, đường cao AH. Vẽ đường tròn tâm H bán kính HA. D là điểm nằm trên đường tròn (H). Gọi M, N lần lượt là trung điểm của DB và DC. Chứng minh rằng bốn điểm D, M, H, N cùng thuộc một đường tròn. Hướng dẫn Gọi E là giao điểm của DH và (H) thì 2 . .BH HC AH HD HE  nên DBEC nội tiếp. BED BCD  MHD MND  Bài 45. Hsg TP HN. Cho tam giác ABC có ba góc nhọn, nội tiếp đường tròn (O) và AB < AC. Các đường cao AD, BE, CF của tam giác ABC cắt nhau tại H. Gọi I là giao điểm của hai đường thẳng EF và CB. Đường thẳng AI cắt (O) tại M (M khác A). a) Chứng minh năm điểm A, M, F, H, E cùng nằm trên đường tròn. b) Gọi N là trung điểm BC, chứng minh ba điểm M, H, N thẳng hàng. c) Chứng minh BM.AC + AM.BC = AB.MC Hướng dẫn a) Dễ dàng nhận thấy tứ giác AFHE nội tiếp, tức A, F, H, E đã nằm trên 1 đường tròn. Ta chứng minh tứ giác AMFH, AMFE hoặc MEHF nội tiếp sẽ có đpcm. Ta chứng minh AMFE nội tiếp. Thật vậy, theo dấu hiệu tích . . . IM IA IB IC IF IE Đpcm. b) Ta chứng minh HN, HM cùng vuông góc với AI, khi đó M, H, N thẳng hàng. Thật vậy 0 0 180 90HMA HAE   Sử dụng bài toán cơ bản. Cho tam giác ABC nội tiếp đường tròn (O) có trực tâm H, đường kính AD. Khi đó DH cắt BC tại trung điểm mỗi đường Áp dụng. Nếu HN kéo dài cắt (O) tại D thì A, O, D thẳng hàng. Khi đó NH vuông góc với IA A B C F E I M H N
  • 18. Luyện thi học sinh giỏi, thi chuyên toán lớp 10 Thầy Hồng Trí Quang 18 Vậy HM, HN cùng vuông góc với IA nên H, M, N thẳng hàng c) Sử dụng định lí P tô lê mê (xem chi tiết về Định lí P tô lê mê trong chuyên đề tứ giác nội tiếp) Bài 46.Chuyên toán ĐHSPHN 2008 - 2009 Cho tam giác ABC vuông tại C. Trên cạnh AB lấy điểm M tùy ý (M khác A và B). Kí hiệu 1 2; ;O O O lần lượt là tâm của các đường tròn ngoại tiếp các tam giác ABC, AMC và BMC. a) Chứng minh bốn điểm 1 2; ; ;C O M O cùng nằm trên một đường tròn ( ) b) Chứng minh O cũng nằm trên đường tròn ( ) c) Xác định vị trí điểm M để đường tròn ( ) có bán kính nhỏ nhất HD c) Gọi R là bán kính của (C) thì 𝐶𝑂 ≤ 2𝑅. Khi đó M là hình chiếu của C trên AB Bài 47.(Chuyên KHTN 2002 – 2003) Cho hình vuông ABCD, M là điểm thay đổi trên cạnh BC (M không trùng với B), N là điểm thay đổi trên cạnh CD (N không trùng với D) sao cho: MAN MAB DAN  a) BD cắt AN và AM tương ứng tại P và Q. Chứng minh rằng năm điểm P, Q, M, C, N cùng nằm trên một đường tròn. b) Chứng minh rằng đường thẳng MN luôn tiếp xúc với một đường tròn cố định khi M, N thay đổi c) Kí hiệu 1S là diện tích tam giác APQ, 2S là diện tích tứ giác PQMN. Chứng minh rằng tỉ số: 1 2 S S không đổi khi M và N thay đổi. d) Tìm GTNN của MN HD ADNQ nội tiếp; các điểm cùng thuộc đường tròn đk MN; b) (A; AD); Tỉ số diện tích bằng bình phương đồng dạng; đs 1d) Tính    2 2 2 2 2 2MN CN CM CN CM    Bài 48.Cho tam giác nhọn ABC (AB < AC), hai đường cao BD và CE cắt nhau tại H. Gọi I là trung điểm của BC, đường tròn qua B, E, I và đường tròn đi qua C, D, I cắt nhau tại K (K khác I) a) Chứng minh năm điểm A, E, H, K, D nằm trên cùng một đường tròn b) *Đường thẳng DE cắt BC tại M. Chứng minh ba điểm M, H, K thẳng hàng.
  • 19. Luyện thi học sinh giỏi, thi chuyên toán lớp 10 Thầy Hồng Trí Quang 19 HD a) AEKD nt, AEHD nt; b) A, K, I thẳng hàng; ICK IAC ; MEKC nội tiếp; 0 90MKI BEC  Bài 49.Cho tam giác ABC không cân, nội tiếp đường tròn (O). Gọi CD là đường kính của đường tròn, qua D kẻ tiếp tuyến với đường tròn cắt đường thẳng AB tại E, EO cắt cạnh BC, CA tại M và N tương ứng. Gọi I là trung điểm AB. Chứng minh rằng a) Bốn điểm O, D, E, I nằm trên một đường tròn b) *O là trung điểm MN HD a) Đường tròn đk OE; b) Từ A kẻ đường thẳng song song OE cắt BC tại F; tứ giác AIDJ nt; IJ song song BC nên JA = JF, đpcm. Bài 50. Cho tam giác nhọn ABC (AB < AC), hai đường cao BD và CE cắt nhau tại H (D trên AC, E trên AB). Gọi I là trung điểm BC, đường tròn qua B, E, I và đường tròn qua C, D, I cắt nhau tại K (K khác I). a) Chứng minh rằng năm điểm A, E, H, K, D nằm trên một đường tròn và 𝐵𝐷𝐾̂ = 𝐶𝐸𝐾̂ b) Đường thẳng DE cắt BC tại M. Chứng minh ba điểm M, H, K thẳng hàng. HD chứng minh 2 tứ giác AEKD, AEHD nội tiếp b) 5 điểm A, E, H, K, D nằm trên đường tròn đường kính AH nên HK AI (1) 0 180AKE EKI  nên A, K, I thẳng hàng. ICK DEK nên MEKC nội tiếp MEC MKC  Mặt khác MEB AED 0 90MKI BEC   MK AI  (2) Từ (1) và (2) thì M, H, K thẳng hàng Bài 51.Cho nửa đường tròn tâm O, đường kính AB, M là điểm đối xứng của O qua A. Đường thẳng qua M cắt nửa đường tròn (O) tại C và D (C nằm giữa M và D). Gọi E là giao điểm của AD và BC. Chứng minh rằng 3 BC AE AD BE  Bài 52.Cho tam giác ABC. Các điểm D, E di động trên các tia BA, CA sao cho 3BD = 2CE. a) Vẽ (O’) ngoại tiếp tam giác ABC. Gọi M là điểm chính giữa cung BC (M và A nằm khác phía đối với BC). I là điểm trên cạnh BC và 3 2BI IC , MI cắt (O’) tại N khác M. Chứng minh rằng tâm O của đường tròn ngoại tiếp tam giác ADE thuộc một đường thẳng cố định. Bài 53.Từ điểm A nằm ngoài đường tròn (O) vẽ các tiếp tuyến AM, AN đến đường tròn (O) và cát tuyến ADC. B là điểm trên cung CM (không chứa D). Gọi H là giao điểm của MN và BD, E là giao điểm của CH và đường tròn (O). Chứng minh rằng A, E, B thẳng hàng.
  • 20. Luyện thi học sinh giỏi, thi chuyên toán lớp 10 Thầy Hồng Trí Quang 20 Bài 54.Cho đường tròn (O) có dây cung BC (khác đường kính) cố định, A là điểm chuyển động trên cung lớn BC, M là trung điểm dây BC. Gọi D là giao điểm của AM và cung nhỏ BC, N là giao điểm của AB và CD. Chứng minh rằng N thuộc một đường thẳng cố định. Bài 55.*Cho tam giác ABC vuông tại A có AM là đường trung tuyến, AH là đường cao. Trên tia đối của tia AM lấy điểm P (P khác A). Các đường thẳng qua H vuông góc với AB và AC lần lượt cắt các đường thẳng PB và PC tại Q và R tương ứng. Chứng minh A là trực tâm tam giác PQR Bài 56.*Cho tam giác ABC nhọn, ba đường cao AD, BE và CF cắt nhau tại H. Qua A vẽ các đường thẳng song song với BE, CF lần lượt cắt các đường thẳng CF, BE tại P và Q. Chứng minh rằng PQ vuông góc với trung tuyến AM của tam giác ABC. ỨNG DỤNG TỨ GIÁC NỘI TIẾP Ứng dụng hệ thức để chứng minh đẳng thức Hệ thức 1. Cho đường tròn (O;R) và điểm A nằm ngoài đường tròn. Kẻ tiếp tuyến AB (B là tiếp điểm) và các cát tuyến ACD, AEF với đường tròn. Chứng minh rằng 2 2 2 . .AFAB AC AD AE AO R    (*) Hệ thức 2. Cho đường tròn (O;R) và một điểm A nằm trong đường tròn  A O . Qua A kẻ hai dây cung CD và EF. Chứng minh rằng 2 2 . .AFAC AD AE R OA   (**) Các hệ thức (*) và (**) được gọi là hệ thức lượng giác trong đường tròn. Bạn đọc có thể chứng minh chúng bằng kiến thức của tam giác đông dạng. vận dụng hệ thức lượng đó ta sẽ giải quyết được nhiều bài toán về chứng minh đẳng thức và bất đẳng thức trong đường tròn. Sau đây là một thí dụ minh họa. Bài 57. Cho tam giác ABC với I, J lần lượt là tâm đường tròn nội tiếp, tâm đường tròn bàng tiếp góc A. chứng minh rằng .AJ .AI AB AC . Lời giải. Ta có ICJ 90o IBJ   A J B C I O D
  • 21. Luyện thi học sinh giỏi, thi chuyên toán lớp 10 Thầy Hồng Trí Quang 21 Suy ra tứ giác IBJC nội tiếp đường tròn đường kính IJ, tâm O (trung điểm IJ). Trên tia AB lấy điểm D sao cho AD AC . Tam giác ACD cân tại A nên AJ là trung trực của CD, suy ra OD OC . Vậy D thuộc đường tròn (O). Áp dụng hệ thức (*) ta có .AJ .AI AD AB , mà AD AC nên .AJ .AI AB AC (đpcm) Bài 58. Cho tam giác ABC với hai đường phân giác trong và ngoài của góc BAC lần lượt là AD và AE. Chứng minh rằng 2 2 . . D .AB AC DB DC A EB EC AE    Lời giải. Giả sử O là tâm của đường tròn ngoại tiếp tam giác ABC. Tia AD cắt đường tròn ở M (điểm chính giữa cung BC). Tia đối của tia AE cắt đường tròn tại N. dễ thấy 90o MAN  nên MN là đường kính của đường tròn (O). Suy ra MN BC . Xét hai tam giác AMB và ADC có 1 2 ( ),A A gt AMB ACD  (góc nội tiếp cùng chắn cung AB), suy ra AMB ACD  (g.g) Nên . . AB AM AB AC AM AD AD AC    hay   2 . . .AB AC AD DM AD AD DM AD    Áp dụng hệ thức (***) với hai dây cung AM và BC ta có . .DM AD DB DC . Do đó 2 . . DAB AC DB DC A  (1) Hai tam giác ANB và ACE có 3 4A A (cùng phụ với hai góc bằng nhau 1 2A A );  ABN AEC AMN  , suy ra ANB ACE  (g.g) nên AB AN AE AC    2 . . . . . AB AC AN AE EN EA AE AB AC EN AE AE        Áp dụng hệ thức (*) với hai cát tuyến EAN và ECB ta có . .EN AE EB EC , do đó 2 . .AB AC EB EC AE  (2) Kết hợp (1) và (2) ta có đpcm 4 2 1 3 D E N M B A C
  • 22. Luyện thi học sinh giỏi, thi chuyên toán lớp 10 Thầy Hồng Trí Quang 22 Bài 59. Hệ thức Euler. Cho tam giác ABC nội tiếp đường tròn (O;R). Gọi I là tâm và r là bán kính đường tròn nội tiếp, J là tâm và or là bán kính đường tròn bàng tiếp góc A của tam giác ABC. Chứng minh rằng 2 2 2OI R Rr  và 2 2 2 oOJ R Rr  . Lời giải. Theo thí dụ 1, ta có tứ giác IBJC nội tiếp đường tròn đường kính Ị, tâm M là trung điểm IJ đồng thời là điểm chính giữa của cung nhỏ BC của đường tròn (O) nên MB MI MJ  Gọi D và E lần lượt là hình chiếu vuông góc của I và J xuống đường thẳng AB, còn MN là đường kính của đường tròn (O). Xét hai tam giác vuông IAD và MNB có 1A N (góc nộ tiếp cùng chắn cung MB) nên đồng dạng, suy ra IA ID MN MB  . . . 2IAMB IAMI ID MN Rr    Áp dụng hệ thức (**) đối với dây cung AIM ta có 2 2 .IA IM R OI  , suy ra 2 2 2R OI Rr  hay 2 2 2OI R Rr  Tương tự, hai tam giác vuông JAE và MNB đồng dạng nên ta có JA JE MN MB  hay . . 2 . a JA JE JA MJ JE MN R r MN MJ     . Áp dụng hệ thức (*) đối với cát tuyến JMA, ta có 2 2 .JA JM OJ R  . Từ đó suy ra 2 2 2 2 2 . 2 .a aOJ R R r OJ R R r     Tự luyện Bài 60. Cho tam giác ABC (các góc B, C đều nhọn), các đường cao BD, CE cắt nhau tại H. chứng minh rằng 2 . .BH BD CH CE BC  D E N M O B A C J
  • 23. Luyện thi học sinh giỏi, thi chuyên toán lớp 10 Thầy Hồng Trí Quang 23 Bài 61. Cho M là một điểm tùy ý thuộc đường thẳng cố định d nằm ngoài đường tròn (O;R). Từ M kẻ hai tiếp tuyến MP và MQ với đường tròn (O) trong đó P, Q là các tiếp điểm. Hạ OH vuông góc với đường thẳng d. Dây cung PQ cắt OH ở I, cắt OM ở K. Chứng minh rằng a) 2 . .OI OH OK OM R  . b)Khi M thay đổi trên đường thẳng d thì vị trí của điểm I luôn cố định. Bài 62. Từ một điểm M nằm ngoài đường tròn (O;R) vẽ hai tiếp tuyến MA, MB (A, B là tiếp điểm) và một cát tuyến qua M cắt đường tròn tại C, D (C nằm giữa M và D). Gọi E là giao điểm của AB và OM. Khi cung CAD nhỏ hơn cung CBD. Chứng minh rằng 2DEC DBC . Lời giải. Áp dụng hệ thức (*) ta có 2 .MB MC MD . Trong tam giác vuông OBM có BE là đường cao nên 2 . . .MB ME MO MC MD ME MO   . Suy ra ( . . )MEC MDO c g c MEC MDO    tứ giác CDOE nội tiếp DOC DEC  . Mà 2DOC DBC (cùng chắn cung DC của đường tròn (O)) nên 2DEC DBC . Bài 63. Cho đường tròn (O) đường kính AB. Vẽ đường tròn tâm A cắt đường tròn (O) ở C và D. Kẻ dây BN của đường tròn(O), cắt đường tròn (A) tại điểm E ở bên trong đường tròn (O). Chứng minh rằng 2 .NE NC ND . Bài 64. Cho tứ giác ABCD. Các đường thẳng AD, BC và AB, CD lần lượt cắt nhau tại E và F. Chứng minh rằng điều kiện cần và đủ để tứ giác ABCD nội tiếp đường tròn là 2 . .EA ED FA FB EF  . Lời giải. E C M O A B D
  • 24. Luyện thi học sinh giỏi, thi chuyên toán lớp 10 Thầy Hồng Trí Quang 24 Giả sử đường tròn ngoại tiếp tam giác ABE cắt EF tại M, đường tròn ngoại tiếp tam giác ADF cắt EF tại N. Áp dụng hệ thức (*) với hai cát tuyến EAD, ENF và hai cát tuyến FAB; FMN ta có . .EF . . EA ED EN FA FB FM EF   (1) (2) Từ (1) và (2) có  . .EA ED FA FB EF EN FM   (3) Giả sử tứ giác ABCD nội tiếp, ta có ABC ADF (cùng bù với góc ADC ), mà ABC AME (cùng bù với góc ABE ) nên AME ADF . Suy ra tứ giác AMFD nội tiếp, chứng tỏ M N . Từ (3) suy ra 2 . . EFEA ED FA FB  (4) Ngược lại, giả sử có (4), kết hợp với (3), suy ra EF EN FM  . Chứng tỏ M N . Từ đó ;AME ABC AME ADF  , suy ra ADF ABC nên 180o ADC ABC  hay tứ giác ABCD nội tiếp đường tròn. Định lí P tô lê mê và ứng dụng Định lí Ptô-lê-mê có thể phát biểu thành định lý thuận và đảo: -Thuận:Nếu một tứ giác nội tiếp trong một đường tròn thì tích của hai đường chéo bằng tổng các tích của các cặp cạnh đối diện. - Đảo:Nếu một tứ giác thỏa mãn điều kiện tổng các tích của các cặp cạnh đối diện bằng tích của hai đường chéo thì tứ giác đó nội tiếp một đường tròn. *] Ứng dụng định lí Ptoleme để chứng minh đẳng thức Bài 65. Cho tam giác đều ABC nội tiếp đường tròn tâm O bán kính R. Gọi M là điểm bất kì thuộc cung BC C E F D B M A N
  • 25. Luyện thi học sinh giỏi, thi chuyên toán lớp 10 Thầy Hồng Trí Quang 25 a) Chứng minh rằng MA = MB + MC b) Gọi D là giao điểm của MA và BC. Chứng minh rằng: 1 1 1 MB MC MD   c) * Tính tổng 2 2 2 MA MB MC  theo R. HD b) MCD đồng dạng MAB; c) Đặt MA = x, MB = y, Tổng = 2 2 2 2( )x y xy  Kẻ BH vuông góc AM; 2MH = y nên 2 2 2 2 2 2 2 2 2( ) 2( ) 6RAB AH BH x y xy      (Đề thi vào trường THPT chuyên Lê Quí Đôn, Quảng Trị, năm học 2005-2006) Bài 66. Cho tam giác đều ABC có các cạnh bằng a.Trên AC lấy điểm Q di động, trên tia đối của tia CB lấy điểm P di động sao cho 2 .AQ BP a . Gọi M là giao điểm của BQ và AP. Chứng minh rằng: AM MC BM  Chứng minh: Từ giả thiết 2 . . .ABAQ BP a AQ BP AB   . Xét ΔABQ  ΔBPA(c.g.c) ABQ=APB (1) Lại có ABQ+MBP=600 (2) Từ: (1), (2) ⇒BMP=1800 − MBP−MPB =1200 AMB=1800 −BMP=1800 −1200 = 600 =ACB.  tứ giác AMCB nội tiếp được đường tròn. Áp dụng định lí Ptô-lê-mê cho tứ giác AMCB nội tiếp và giả thiết AB = BC = CA ta có: . . .AB MC BC AM BM AC AM MC BM     (đpcm) Bài 67. Tam giác ABC vuông có BC > CA > AB. Gọi D là một điểm trên cạnh BC, E là một điểm trên cạnh AB kéo dài về phía điểm A sao cho BD = BE = CA. Gọi P là một điểm trên cạnh AC sao cho E, B, D, P nằm trên một đường tròn. Q là giao điểm thứ hai của BP với đường tròn ngoại tiếp  ABC. Chứng minh rằng: AQ + CQ = BP (Đề thi chọn đội tuyển Hồng Kông tham dự IMO 2000, HongKong TST 2000) Dựa vào các đại lượng trong tam giác bằng nhau theo giả thiết, ta sử dụng ĐL tam giác đồng dạng để suy ra các tỉ số liên quan và sử dụng phép thế để suy ra điều phải chứng minh. Chứng minh: Xét các tứ giác nội tiếp ABCQ và BEPD ta có: CAQ=CBQ=DEP (cùng chắn các cung tròn)
  • 26. Luyện thi học sinh giỏi, thi chuyên toán lớp 10 Thầy Hồng Trí Quang 26 Mặt khác AQC=1080 −ABC=EPD Xét ΔAQC và ΔEPDcó: AQC=EPD; CAQ=DEP  ΔAQCΔEPD  AQ/EP=CA/ED  AQ.ED =EP.CA=EP.BD( 1) (do AC=BD) AC/ED=QC/PD  ED.QC=AC.PD=BE.PD (2) (do AC=BE) Áp dụng định lí Ptô-lê-mê cho tứ giác nội tiếp BEPD ta có: EP.BD+BE.PD=ED.BP (3) Từ (1), (2), (3) suy ra: AQ.ED + QC.ED = ED.BP  AQ+QC=BP (đpcm) Bài 68. Định lí Carnot Cho tam giác nhọn ABC nội tiếp trong đường tròn (O,R) và ngoại tiếp đường tròn (I,r). Gọi x, y, z lần lượt là khoảng cách từ O tới các cạnh tam giác. Chứng minh rằng: x y z R r    Chứng minh: Gọi M,N,P lần lượt là trung điểm của BC,CA,AB. Giả sử x = OM, y = ON, z = OP, BC = a, CA = b, AB = c. Tứ giác OMBP nội tiếp, theo đẳng thức Ptô-lê-mê ta có: . . .OB PM OP MB OM PB  Do đó: 2 2 2 . . 1(. )Rb z a x c  Tương tự ta cũng có 2 2 2 . . .R c y a xb  (2) và 2 2 2 . . .R a y c z b  (3) Mặt khác: 2 2 2 2 2 2 ( ) . .r a b c SABC SOBC SOCA SOAB x a yb z c         (4) Từ (1), (2), (3), (4) ta có: 2 2 ( )( ) ( )( )R r a b c x y z a b c            R r x y z (Bài này còn gọi là Định lí Carnot- 1 định lí khá là quen thuộc và cách chứng minh cũng khá đơn giản. Ứng dụng của định lí này như đã nói là dùng nhiều trong tính toán các đại lượng trong tam giác) Ứn dụng Carnot: Bài 69. *Ứng dụng Định lí Carnot Cho tam giác ABC cân tại A, nội tiếp trong đường tròn (O) bán kính R = 1, có AB = 2.BC. Tính bán kính đường tròn nội tiếp (O’) nội tiếp tam giác ABC?
  • 27. Luyện thi học sinh giỏi, thi chuyên toán lớp 10 Thầy Hồng Trí Quang 27 Sử dụng tam giác đồng dạng, r = 3/8 R + r =OI + OK + OH (khoảng cách đến 3 canh). Sử dụng diện tích; Chứng minh AO’ = 4r bằng tam giác đồng dạng; OI, OK, OH vuông Bài 70. Cho tam giác ABC nội tiếp trong đường tròn (O) và AC=2AB. Các đường thẳng tiếp xúc với đường tròn (O) tại A,C cắt nhau ở P. Chứng minh rằng BP đi qua điểm chính giữa của cung BAC. Chứng minh: Gọi giao điểm của BP với đường tròn là N. Nối AN,NC. Xét NPCvà CPB có: PCNˆ=PBCˆ,Pˆ chung  ΔNPC  ΔCPB(g.g)  PC/PB=NC/BC(1) Tương tự ta cũng có ΔPAN  ΔPBA(g.g)  AP/BP=AN/AB (2) Mặt khác PA=PC (2)  PA/PB=NC/BC=AN/AB  NC.AB=BC.AN(3) Áp dụng định lí Ptô-lê-mê cho tứ giác nội tiếp ABCN ta có: AN.BC+AB.NC=AC.BN Từ (3) 2AB.NC=AC.BN=2AB.BN NC=BN. (đpcm) Bài 71. Cho tam giác ABC có I là tâm đường tròn nội tiếp, O là tâm đường tròn ngoại tiếp và trọng tâm G. Giả sử rằng 𝑂𝐼𝐴̂ = 900 . Chứng minh rằng IG song song với BC. Chứng minh Kéo dài AI cắt (O) tại N. Khi đó N là điểm chính giữa cung BC (không chứa A). Ta có: BN=NC(1). Lại có:
  • 28. Luyện thi học sinh giỏi, thi chuyên toán lớp 10 Thầy Hồng Trí Quang 28 IBNˆ= BINˆ BN=IN(2) Do OI  AI suy ra IA=IN=1/2 sđ cung BC(3) Từ (1),(2),(3) BN=NC=IN=IA(4) Áp dụng định lí Ptô-lê-mê cho tứ giác nội tiếp ABNC ta có: BN.AC+AB.NC=BC.AN Từ (4) BN(AC+AB)=2BN.BC AC+AB=2BC (5) Áp dụng tính chất đường phân giác trong tam giác và (5) ta có: AB/BD=IA/ID=AC/CD=(AB+AC)/(BD+CD)=(AB+AC)/(BC)=2BC/BC=2 Vậy IA/ID=2(6) Mặt khác G là trọng tâm của tam giác suy ra AG/GM=2(7) Từ (6),(7) IA/ID=2=AG/GM Suy ra IG là đường trung bình của tam giác ADM hay IG song song với BC. (đpcm) Bài 72. Cho tam giác ABC nội tiếp đường tròn (O), CM là trung tuyến. Các tiếp tuyến tại A và B của (O) cắt nhau ở D. Chứng minh rằng: ACD BCM Chứng minh: Gọi N là giao điểm của CD với (O). Xét tam giác DNB và DBC có: DBNˆ=DCBˆ,Dˆ chung. ΔDBN  ΔDCB(g.g) NB/CB=BD/CD(1) Tương tự ta cũng có : ΔDNA  ΔDAC(g.g) NA/AC=DA/CD(2) Mà BD=DAnên từ (1),(2)NB/CB=NA/AC  NB.AC=AN.BC(3) Áp dụng định lí Ptô-lê-mê cho tứ giác nội tiếp ANBC ta có: AN.BC+BN.AC=AB.NC Từ (3) và giả thiết AB=2BM 2AN.BC=2BM.NC AN/NC=BM/BC
  • 29. Luyện thi học sinh giỏi, thi chuyên toán lớp 10 Thầy Hồng Trí Quang 29 Xét ΔBMC và ΔNAC có: MBCˆ=ANCˆ, AN/NC=BM/BC ΔBMC ΔNAC(c.g.c) BCMˆ=NACˆ Vậy bài toán được chứng minh. Ứng dụng định lí Ptoleme để chứng minh đẳng thức Bài 73. Cho tứ giác nội tiếp có các cạnh liên tiếp bằng a, b, c, d và các đường chéo bằng p, q. Chứng minh rằng:   2 2 2 2 pq a b c d   HD Bài toán đơn giản, sử dụng Ptoleme và Bunhia Cho đường tròn (O)) và BC là một dây cung khác đường kính của đường tròn. Tìm điểm A thuộc cung lớn BC sao cho AB + AC lớn nhất. Gọi D là điểm chính giữa của cung nhỏ BC thì DB = DC = a không đổi. Áp dụng Ptoleme thì A là là điểm chính giữa cung lớn BC. Tự luyện Bài 74. (149T5/246 – 45 năm).Cho tam giác ABC, biết rằng: 𝐴̂ = 2𝐵̂ = 4𝐶̂. Chứng minh rằng: 1 𝐴𝐵 = 1 𝐵𝐶 + 1 𝐴𝐶 HD: Trên cung BC lấy D sao cho hai cung AB = BD. Áp dụng định lí Ptoleme Bài 75. Cho tứ giác ABC nội tiếp đường tròn (O). Chứng minh: . . . . AC BC CD AB BD BD BC BA DC DA    Lấy E, F thuộc đường tròn sao cho: ;CDB ADE BDA DCF  Áp dụng định lí cho tứ giác AECD và BCDF. Chứng minh ED = FC (do 2 cung bằng nhau) ta có đpcm Bài 76. Cho tam giác ABC với BE, CF là các đường phân giác trong. Các tia EF, FE cắt đường tròn ngoại tiếp tam giác theo thứ tự tại M và N. Chứng minh rằng: 1 1 1 1 1 1 BM CN AM AN BN CM     
  • 30. Luyện thi học sinh giỏi, thi chuyên toán lớp 10 Thầy Hồng Trí Quang 30 HD Đặt BC = a, CA = b, AB = c. áp dụng đli cho tứ giác AMBC, ANCB. Mặt khác ; AM MF AN AF BN BF BM MF   Theo tính chất phân giác ta có: . . AM AN AF b BM BN BF a   Tương tự: . . AM AN AE c BM BN CE a   Bài 77. Giả sử M, N là các điểm nằm trong tam giác ABC sao cho ;MAB NAC MBA NBC  . Chứng minh rằng: . . . 1 . . .CB AM AN BM BN CM CN AB AC BA BC CA    HD lấy K trên đường thẳng BN sao cho BCK BMA ; AB BK MB BC   Mặt khác: AB BK AK MB BC CM   Từ đó tính được CK, AK, BK. Thay vào định lí Ptoleme trong tứ giác ABCK ta có: . . .AC NK AN CK CN AK  . .( ) . . . AB BC AC BK BN AN CK CN AK AC BN BM         
  • 31. Luyện thi học sinh giỏi, thi chuyên toán lớp 10 Thầy Hồng Trí Quang 31 Bài 78. (CMO 1988, Trung Quốc) ABCD là một tứ giác nội tiếp với đường tròn ngoại tiếp có tâm ) và bán kính R. Các tia AB, BC, CD, DA cắt (O,2R) lần lượt tại A′, B′, C′, D′. Chứng minh rằng: ( )2A B B C C D D A AB BC CD DA            Bài 79. Cho đường tròn (O) và dây cung BC khác đường kính. Tìm điểm A thuộc cung lớn BC của đường tròn để AB+2AC đạt giá trị lớn nhất. Bài 80. Cho tam giác ABC nội tiếp đường tròn (O). Đường tròn (O′) nằm trong (O) tiếp xúc với (O) tại T thuộc cung AC (ko chứa B). Kẻ các tiếp tuyến AA′,BB′,CC′tới (O′). Chứng minh rằng: BB′.AC=AA′.BC+CC′.AB Ứng dụng tứ giác nội tiếp Chứng minh ba điểm thẳng hàng Updating… Chứng minh dựa vào góc Chứng minh dựa vào song song Bài 81. Cho tam giác ABC nội tiếp đường tròn (O) và I là tâm đường tròn nội tiếp. Gọi M, N, D là điểm chính giữa các cung nhỏ BC, CA, AB. Giả sử MN cắt AC tại Q; MD cắt AB tại P. Chứng minh rằng: a) MB = MI b) Ba điểm P, I, Q thẳng hàng. HD a) nếu I thuộc AD thì I là tâm nội tiếp khi và chỉ khi MB = MI. b) Từ câu a suy ra MP là trung trực BI, suy ra góc PBI = góc PIB, suy ra IP song song BC. Tương tự IQ song song BC Ứng dụng tứ giác nội tiếp Chứng minh ba đường đồng quy Updating…
  • 32. Luyện thi học sinh giỏi, thi chuyên toán lớp 10 Thầy Hồng Trí Quang 32